Vous êtes sur la page 1sur 41

Lesson 2, Module 1

The tertiary structure of protein is formed mainly due


to disulfide bonds between side radicals of one amino
acid, only. Point out it:
A. Cys -------B. Met
C. Asp
D. Lys
E. His
Primary structure of proteins is formed due to one
type of bonds. Point out it:
A. Peptide bond ------------B. Disulfide bond
C. Ester bond
D. Hydrogen bond
E. Metal bond
Polypeptide chains of collagen include specific amino
acids. Name one of them:
A. Hydroxyproline -----------------B. Formyl-methyonine
C. Cysteine
D. -alanine
E. Ornithine
There are many important protein functions in the
human organism. Point out that of them, which isn't
peculiar for proteins:
A. Catalyst
B. Transfer of substances
C. Antibody
D. Structural component of a cell
E. Solvent ----------------The solubility of proteins in saline solutions is
determined by their native structure. Point out the
protein, which will swell only in saline solution:
A. Elastin
B. Albumin
C. Myoglobin
D. Immunoglobulin
E. Pepsin
The proteins are able to carry out the regulatory
function. Find out those proteins:
A. Aminopeptidase
B. Insulin -------------------C. Collagen
D. Hemoglobin
E. Immunoglobulin G
All proteins are divided into simple and conjugated
ones. Find out the simple proteins among these ones:
A. Albumin of egg
B. Histone
C. Globulin of egg
D. Protamine ---------------------E. All the proteins above
The molecule of simple protein insulin contains two
polypeptide chains. Specify the level of protein
organization, at which the insulin is able to act as
hormone:
Quaternary
Tertiary

Primary
Secondary
A conformation after limited proteolysis
Choose the factor that causes the sedimentation of
protein in solution without denaturation:
Ammonia sulfate -----------Toluene
Sulfuric acid
Nitric acid
Sodium hydroxide
Histones are related to basic proteins. That is because
there is high content of basic amino acid residues in
their structure. Point out these amino acids:
lanine, Glycine
Asparagine, Glutamine
Arginine, Lysine -------------------Leucine, Valine
Tryptophan, Tyrosine
The mixture of proteins can be separated by salting-out.
Specify the reagent formula that is used for this
purpose:
HNO3
NaCl --------------H3PO4
C2H5OH
C6H6
Point out the type of bond allowing the formation of
alpha-helix structure:
Ester bond between side chain radicals
Hydrogen bonds between peptide fragments -------------Disulfide bond between two cysteine radicals
Electrostatic interaction
Hydrogen bonds between side-chain radicals
Protein properties may be changed under the influence
of some factors. Find out them:
Strong alkaline medium
Organic solvent
The temperature of the environment
Strong acidic medium
All the factors placed above ------------------The polypeptide chain gets the globular structure after
the formation of various bonds between the radicals of
amino acid residues. Specify the strongest bond in the
globular structure:
Disulfide bond
Hydrogen bond
Donor-acceptor bond
Electrostatic interaction
Hydrophobic interaction
The isoelectric point (I.P.) of a protein depends upon
the amino acids composition of the protein. Choose the
amino acid which high content decreases the I.P. value
of protein:
lanine
Aspartic acid
Histidine
Leucine
Tryptophan
Proteins are obligatory components of human diet.
Specify the function of proteins in this case:
Nutritive
Transport
Regulatory

Structural
Catalytic
Different functional groups may be found in the
structure of L-amino acid residues of proteins. Identify
the group that is able to form ester bond:
CH3
SH
CONH2
OH -------------------NH2
The isoelectric point of simple protein equals 7.2.
Propose the pH of buffer solution used for the
electrophoresis method to separate this protein from the
mixture with a condition to leave it on the start line of
carrier:
pH=7.0
pH=7.6
pH=7.4
pH=5.0
pH=7.2
All are aromatic amino acids EXCEPT:
A. Phenylalanine
B. Tyrosine
C. Tryptophan
D. Lysine ----------------E. Positions A and D are correct
Ninhydrin is a reagent to prove the presence of alphaamino group in the structure of amino acid due to
change of its color (violet color is observed). Choose the
amino acid whose solution changes the color of this
reagent in other way it becomes yellow:
A. L-methionine
B. L-tyrosine
C. L-serine
D. L-proline -----------------------E. L-alanine
The content of these amino acids in the composition of
acidic protein pepsin is too big in comparison with the
content of others amino acids in it. Name them:
Lysine and Arginine
Valine and Leucine ------------------Aspartic acid and Glutamic acid
Alanine and Glycine
Tyrosine and Tryptophan
The yellow color sediment appearance after the addition
of strong nitric acid to albumin solution is due to the
presence of aromatic acid residues in protein
composition. Choose those one:
A. L-methionine
B. L-tyrosine ----------------------C. L-serine
D. L-proline
E. L-alanine
What amino acid high content presence in the
composition of polypeptide chain does not allow the
formation of alpha-helical structure as secondary level
of organization?
Proline --------------------Alanine
Glycine
Serine

Threonine
What type of amino acids mainly is represented as
residues in proteins of human body?
L--amino acids
D--amino acids
D--amino acids
L--amino acids ------------------D--imino acids
The structural proteins are involved in maintaining the
shape of a cell or in the formation of matrices in the
body. Point out shape of these proteins:
A. Globular
B. Fibrous ------------------C. Stretch of beads
D. Planar
E. All of above
Which of the following bonds are intact during
denaturation of proteins:
A. Hydrophobic bonds
B. Hydrogen bonds
C. Peptide bonds ----------------D. Ionic bonds
E. All positions are correct
Alpha helix and Beta pleated sheet are examples of:
A. Primary structure
B. Secondary structure ---------------------C. Tertiary structure
D. Quaternary structure
E. All positions are wrong
Biuret test is mainly done for:
Polysaccharides
Proteins ----------------------Lipids
Dipeptides
Any of the above
Half saturation test using salting-out is done for:
Albumin
Globulin -------------------Fibrin
Prothrombin
Haemoglobin
What class of proteins Albumins and Globulins are
related to?
A. Simple proteins ---------------------B. Glycoproteins
C. Chromoproteins
D. Metalloproteins
E. Lipoproteins
What physical-chemical properties are observed for
fibrous protein only?
Solubility in water
Solubility in lipids
Amphoteric properties
Elasticity ------------------Denaturation and renaturation
Lesson 3 , Module 1
The conjugated protein necessarily contains special
component as a non-protein part. Choose the substance
that can't carry out this function:

Glucose
HNO3 --------------------Fe 2+
Heme
Phosphate anion
Which method is better suited to separate a mixture of
compounds into its individual components and detects
small amounts (microgram or even picogram) of
material:
Dialysis
Paper chromatography
Ultracentrifugation ---------------Salting out
Spectrophotometry
Point out a possible cause of hypoproteinemia:
Affected liver cells ---------------Multiple myeloma
Decreased permeability of the capillary wall
Overeating
Paraproteinemia
Point out a possible cause of hyperproteinemia
Increased permeability of the capillary wall
Infection (disturbed the macrophage system)----Affected gastrointestinal tract
Nephritic syndrome
Diabetes mellitus
Which method is appropriate for the determination of
total protein content in the blood serum:
Salting out
Foles test
Dialysis
Electrophoresis
Biuretic method ------------------Choose the conjugated protein in possession of following
characteristics: quaternary structure - 4 polypeptide
chains; non-protein part 4 hemes; function oxygen
transport in the blood:
Low Density Lipoprotein
Albumin
Immunoglobulin
Hemoglobin ----------------Ceruloplasmin
What compound serves as non-protein part of
glycoproteins:
Cu 2+
Fe2+
Galactose ----------------Heme
Phospholipid

Thiamine pyrophosphate
ATP
AMP
When the following amino acids are separated by
running them on agarose gel, at pH 7 (electrophoresis
method), which one of them will be slowest to the anodic
end
Glycine
Valine
Aspartic acid
Lysine ---------------------Glutamic acid
Choose, please, the blood serum index used for
estimation of hyperproteinemia state:
The total content of proteins -----------------The content of albumins
The content of acute phase proteins
The amino acid concentration
The total activity of all the enzymes
Proteoglycans are conjugated proteins containing
different polypeptide chains of core protein and glucose
aminoglycan moiety. Choose the class of conjugated
protein that is related to proteoglycan:
Phosphoprotein
Nucleoprotein
Lipoprotein
Glycoprotein ----------------------Chromoprotein
It is in need to use the heme with iron ion for the active
centers of cytochrome oxidase (the key enzyme of tissue
respiration). Name the class of this conjugated protein:
Flavoprotein s
Nucleoproteins
Lipoproteins
Glycoproteins
Chromoproteins ------------------There is the use of electrophoresis for separation of
proteins of blood plasma to prove diagnosis of diseased
persons. Name the property of proteins that is the basis
for the principle of electrophoresis method:
Optical activity
The big mass of the molecule
The ability of swelling
The high viscosity of the solution
The net charge of the molecule -----------------

Which group of proteins being phosphoproteins posses


an activity but being dephosphorylated has lost the
activity:
Hormones
Transfer of lipids
Transfer of vitamins
Enzymes ------------------Carriers through membrane

Albumins of blood plasma are negative charged under


the condition of electrophoresis method duration. What
electrode name have you to choose for albumins
movement in the electric field?
Catode
Anode --------------Silk electrode
Calomel electrode
Carbon electrode

The conjugated protein necessarily contains special


component as a non-protein part. Choose the substance
that can't carry out this function:
Glucose
Salt of Hg2+ -------------------------

The enzyme preparation cytochrome C is used for the


improvement of tissue respiration in newborns at
asphyxia state. Name the class of this conjugated
protein:
Flavoproteins

Nucleoproteins
Lipoproteins
Glycoproteins
Chromoproteins ---------------The diseased person with diagnosis acute kidney
insufficiency is in urological department of hospital.
Choose the method for cleaning of this person`s blood
from low-molecular compounds which can cause toxic
effect in the organism:
Salting-out
Electrophoresis
Dialysis --------------------Hydrolysis
Affine chromotography
The hormone receptors are related to the class of
conjugated proteins. Name it:
Flavoproteins
Nucleoproteins --------------Lipoproteins
Glycoproteins
Chromoproteins
The phosphorylation (the attraction of phosphate group
to the substrate) of polypeptide chain is often used for
stimulation of biological activity of a protein. Name the
class of conjugated protein formed due to
phosphorylation:
Nucleoproteins
Lipoproteins
Phosphoproteins ------------------Chromoproteins
Glycoproteins
All the proteins are divided into simple and conjugated
ones. Find out the conjugated protein among these ones:
A. Egg albumin
B. Histone -------------C. Myoglobin
D. Protamine
E. Egg globulin
Name the location of deoxyribonucleoproteins in
a cell, but not in the nucleus:
A. Lisosome
B. Cytoplasma
C. Mitochondria --------------D. Microsome
E. Endoplasmic reticulum
Flavoproteins are usually catalysts in a cell due to
the presence of special vitamin fragment in their
structure. Name this vitamin:
A. Nicotin amide
B. Folic acid
C. Panthothenic acid
D. Riboflavin --------------E. Ascorbic acid
Ceruloplasmin (the protein of blood plasma) contains
copper ion and therefore has blue color. Name the class
of this protein:
Metalloproteins ----------------Lipoproteins
Phosphoproteins
Chromoproteins
Glycoproteins

The electrophoresis method is used for the separation of


blood plasma proteins. Name the parameter of the
protein molecule that is in need to determine the pH of
buffer solution for separation of proteins in
electrophoresis method:
Isoelectric point -----------------The mass of the molecule
The diameter of the molecule
The solubility in the water
The solubility in lipids
Ultracentrifugation is in need in biochemical
investigations for:
The study of three-dimensional structure of the
molecule
The separation of mixture from lo-molecular
compounds ----------------The receiving of subcellular fractions of a cell
The investigation of chemical composition of organic
compound
The determination of primary structure of proteins
Complex proteins do various functions. Find out the
function of hemoglobin in erythrocytes:
Regulatory function
Catalytic function
Nutrition function
Transport function --------------Protection against viruses
Triacylglycerols (TG) are synthesized in the liver but
are stored in adipose tissue. Name the class of proteins
promoted the transport of TG from the liver to
adipocytes:
Metalloproteins
Lipoproteins --------------Phosphoproteins
Chromoproteins
Glycoproteins
Name the class of proteins used for the formation of
ribosome subunits:
Flavoproteins
Lipoproteins
Phosphoproteins
Ribonucleoproteins --------------Glycoproteins
Toxic affection of liver results in dysfunction of protein
synthesis. It is usually accompanied by the following
kind of dysproteinemia:
Absolute hypoproteinemia*
Relative hypoproteinemia
Absolute hyperproteinemia
Relative hyperproteinemia
Paraproteinemia

Lesson 4 Module 1
Conjugated enzymes contain cofactors in their structure.
Point out the location of vitamin derivative cofactor in
the structure of enzyme:
Active centre ----------Allosteric centre
Hydrophobic fragment of structure
Hydrophilic fragment of structure

Near the metal-ion-cofactor in the structure


Only one factor can influence on the charge of amino
acid radicals in the enzyme active centre. Name it:
Temperature
Pressure
pH medium -------------The presence of a competitive inhibitor
The surplus of a reaction product
One of the important properties of enzymes is their
specificity of action. Check up a type of specificity for
salivary amylase:
Absolute ---------------Absolute group
Absolute relative
Relative group
Stereochemical
Some terms are used for the description of non-protein
part of an enzyme. Point out the term for non-protein
part that easily dissociates from polypeptide chain:
Apoenzyme
Coenzyme -------------Prosthetic group
Cofactor
Metall ions
The change of the temperature of environment from
0C to 38C can cause this effect:
The probability of enzyme-substrate complex
formation is increased
A denaturation of enzymes occurs
The enzyme molecular charge changes
The substrate molecular charge changes
Enzyme action specificity varies
There are some factors influencing enzyme activity.
Point out one of them resulting in complete loss of
enzymatic activity:
Vitamin H
arbon dioxide
T = 100 C ------------------P =101325 Pa
Sodium chloride solution
There are some characteristic sites in the enzyme
structure. Choose the most important site for enzyme
function:
Allosteric centre
Active centre ----------------Cofactor
Apoenzyme
Catalytic site
Point out the factor that can cause the damage of
enzyme function in a cell:
Temperature 37C
The presence of activator of enzyme
pH medium about 7.2
The presence of a product of enzymatic reaction
The surplus of protons in a cell ---------------Enzymes are the catalysts of protein nature. Name the
property of enzymes which is not represented at the
inorganic catalysts:
Ability to the denaturation ------------Wide specificity
Inert to chemical substrates
Big half-life
Ability to lowering the energy to activate the
reaction

The oxidation of a substrate may be catalyzed by


enzyme - flavoprotein that contains FAD as prosthetic
group. Name, please, the vitamin used for this nonprotein part of enzyme formation:
Ascorbic acid
Nicotinamide
Riboflavin ----------------Biotin
Adenosine triphosphate
The enzyme hexokinase can catalyze the conversion of
glucose or fructose in tissues. Find out the type of this
enzyme specificity:
Absolute
Absolute group
Absolute relative
Relative group
Stereochemical
The catalytic site of active center of enzyme is used
for:
Conversion of a substrate in the reaction -----Binding with the substrate
Binding with activator
Binding with inhibitor
Removal of a product of the reaction
There are different cofactors in the structure of
conjugated enzymes but only one is used for transfer
of amine group from amino acid to ketoacid. Name it:
Carboxybiotin
Pyridoxal phosphate
Thiamine pyrophosphate
FAD
NAD
Some terms are used for the description enzyme
components. Point out the term of a protein part of
conjugated enzyme:
A. Apoenzyme -------------B. Coenzyme
C. Prosthetic group
D. Cofactor
E. Metall ions
Oxidoreductase can contain prosthetic group with
vitamin B2. Name it:
A. Retinal
B. Flavin adenine dinucleotide (FAD) ----------C. Nicotinamide adenine dinucleotide (NAD)
D. Pyridoxal phosphate
E. Ascorbic acid
A substrate molecule is destructed upon enzyme
action, and the water is used for the products structure
formation. Name the enzyme class:
A. Oxidoreductase
B. Hydrolase -------------C. Lyase
D. Ligase
E. Isomerase
A qualitative composition of product molecule is
completely identical to substrate one, but the structure
is different. Name the enzyme class:
A. Oxidoreductase
B. Hydrolase
C. Lyase
D. Ligase

E. Isomerase ---------------ATP molecules may be used for Transferases and


Ligases function. Point out the signs of ATP use for
Ligases class:
A. ATP is used for a substrate dephosphorylation
B. ATP is used for a substrate phosphorylation
C. ATP is used for hydrolysis of a substrate bond
D. ATP is used for the new bond formation during
the interaction of two substrates --------------E. ATP is used for a substrate decarboxylation.
Coenzyme forms are correctly matched to vitamins
except one. Point out it:
A. Biotin carboxylated biotin
B. Vitamin B1 - ATP -------------C. Niacin NAD+ + NADP+
D. Vitamin B2 FMN + FAD
E. Pantothenic acid CoASH
Enzymes mediating transfer of a structural fragment
from one molecule to another are:
Transferases -------------------Oxidases
Lyases
Peptidases
Ligases
Which bond is cleaved by Alpha amylase in oral cavity?
Alpha 1-4 glycosidic -----------------Alpha 1-6 glycosidic
Beta 1-4 glycosidic
Beta 1-6 glycosidic
Ester bond in any ester structure
Which of the following is the reaction that is catalyzed
by lyase:
Dehydration
Oxidation
Oxidative decarboxylation
Hydrolysis -------------Acetylation
All biological catalysts are not proteins This
statement is justified by this notion:
All enzymes do not follow the Michaelis Menten
hypothesis
RNAs can act as ribozymes -------------Antibodies take part in the catalysis of many reactions
Metal ions are involved in attachment to enzymes
Enzyme activity may be controlled by hormones
LDH1 and LDH2 levels are raised in the following
organ damage
Heart, RBC, kidney ---------------------Heart, kidney, liver
Liver, brain, kidney
Brain, heart, liver
Brain, bones, liver
Name, please, the principle base used for the
classification of enzymes:
Type of chemical reaction catalyzed by enzyme --------Chemical structure of enzyme
Type of energy conversion
Chemical structure of non-protein part of the enzyme

Chemical structure of products for enzymatic reaction


Name the vitamins whose derivatives are used for the
formation of oxidoreductase structure:
B2, B3
B6, B9
C, P
A, E
H, D
Chymotrypsin is the proteolytic enzyme catalyzing the
cleavage of peptide bonds in any protein molecule.
Name the class of this enzyme:
Oxidoreductase
Isomerase Lyase
Ligase
Hydrolase -------------------The active centre of simple enzyme is composed from:
The cofactor and prosthetic group
Linear fragment of polypeptide chain
Some amino acid residues of polypeptide chain placed
in the same spatial fragment ----------------One polypeptide chain completely
The terminal amino acid residues
The pancreatic amylase is in need to cleave the alpha1.4-glycosidic bonds in the structure of polysaccharides
using the water as the substrate. Specify the class of this
enzyme:
A. Isomerase
B. Ligase
C. Lyase
D. Hydrolase ------------E. Oxido reductase
The relative group specificity may be found for enzymes
catalyzing the digestion of proteins in GIT. Find out
their trivial name:
Protein kinase --------------Protein phosphatase
Peptidase
Transaminase
Urease
Researchers isolated five isozymes of Lactate
dehydrogenase from human blood serum and studied
their properties. What property of these isozymes
indicates that they are genetic forms of the same
enzyme?
They have the same molecular weight
They catalyze the same reaction*
The same tissue localization
The same electrophoretic mobility
The same net charge of the molecule
In case of enterobiasis acrihine - the structural
analogue of vitamin B2 - is administered. The synthesis
disorder of which enzymes does this medicine cause in
microorganisms?
FAD-dependent dehydrogenases ----------------Cytochromeoxidases
Peptidases
NAD-dependet dehydrogenases
Aminotransferases

In clinical practice tuberculosis is treated with izoniazid


preparation that is an anti-vitamin able to penetrate
into the tuberculosis bacillus. Tuberculostatic effect is
induced by the interference with replication processes
and oxidation-reduction reactions due to the buildup of
pseudo-coenzyme:
FMN
NAD
CoQ
FAD
TDP
Lesson 5 Module 1
.
E. Fisher`s theory explains the mechanism of enzyme
action with the fixed type of specificity, only. Name it:
A. Absolute -------------B. Absolute group
C. Absolute relative
D. Relative group
E. Stereochemical
There are some factors influencing enzyme activity.
Point out one of them resulting in complete loss of
enzymatic activity:
A. Vitamin H
B. Oxygen
C. t0 C = 1000 C ---------------D. P =101325 Pa
E. Sodium chloride solution
Choose the factor that changes the cytoplasmic enzyme
conformation mainly:
A. Suicide inhibitor
B. Environmental pH value about 7.4
C. Environmental temperature value about 250 C
D. Allosteric inhibitor ------------------E. Water
Point out the way of proenzyme transformation to the
active enzyme:
A. Limited proteolysis ----------------B. Dehydration
C. Decarboxylation
D. Inhibitor action
E. Vitamin non-protein part dissociation from enzyme
Competitive inhibitor always interacts with enzyme
active centre. Find out the explanation of this
phenomenon:
A. Inhibitor causes the denaturation of active centre
B. Inhibitor is similar to a substrate structure ----------C. Inhibitor is an exact copy of a substrate structure
D. Inhibitor is similar to the product's structure
E. Inhibitor forms a covalent type of bonds with amino
acid residues of active centre
Covalent modification of inactive form of enzyme may
be catalyzed by special enzyme in a cell. Name it:
A. Esterase
B. Ligase
C. Protein kinase ---------------D. Hydroxylase
E. Oxygenase

Succinate dehydrogenase catalyses the dehydrogenation


of succinate. Malonic acid
HOOC-CH2-COOH is used to interrupt the action of
this enzyme. Choose the inhibition type:
Allosteric
Competitive ---------------Non-competitive
Dephosphorylation
Limited proteolysis
Choose the name of scientists whose experiments are
recognized as the basis in understanding of induced
fit theory for mechanism of enzymatic reaction:
Michaelis L.
Menten M.
Koshland D. ---------------Fisher E.
Haldane R.
Diisopropyl phosphofluoride (DFP) reacts with serine
proteases irreversibly and therefore is:
Allosteric inhibitor --------------Non-competitive inhibitor
Competitive inhibitor
Affinity label inhibitor
A stimulator
The common feature of an enzyme-cascade system
regulation is:
A. Feed back inhibition -------------B. Competitive inhibition
C. Counter-regulation
D. Amplification
E. Suicide inhibition
The formation of ES complex is due to various types of
bonds between E and S. Specify the type of bond, which
is usually formed between charged functional groups in
this case:
A. Peptide bond
B. Hydrophobic interaction
C. Hydrogen bond
D. Donor-acceptor bond
E. Electrostatic interaction -------------The common enzymatic reaction may be represented so:
E + S ES ESEP E + P. Try to name using this
equation all the factors that can influence the rate of
this reaction:
A. The concentration of a substrate, only
B. The concentration of enzyme, only
C. The concentration of a substrate, enzyme and
product, only
D. The concentration of enzyme-substrate
complex, only
E. The concentration of a substrate, enzyme,
product and stability of ES-complex -------------Heme synthesis starts from glycine and succinyl-SCoA
interaction with -aminolevulinate synthetase help. It is
inhibited by the terminal metabolic product - heme.
Name the inhibition type:
A. Competitive Inhibition
B. Uncompetitive Inhibition
C. Non-competitive Inhibition
D. Limited proteolysis

E. Feedback Inhibition------------The inhibitor influence on the enzymatic reaction rate is


investigated. The graph dependences V - [S] without the
inhibitor (1) and at the presence of the inhibitor (2) are
constructed.

Name the type of the inhibitor:


A. Competitive
B. Non-competitive --------------C. Uncompetitive
D. Allosteric
E. Complete
The regulation of the enzymatic activity is carried out
by different ways. Point out the way that is used more
often in the regulation of key enzymes:
A. Limited proteolysis
B. Allosteric regulation -----------------------C. Activation by Ca2+
D. The change of pH medium
E. Competitive inhibition
Covalent catalysis as mechanism of enzyme catalysis
was studied in experimental works with one proteolytic
enzyme. Find out it:
A. Pepsin
B. Aldolase
C. Chymotrypsin ---------------D. Decarboxylase
E. Glucoisomerase
Suicide type of inhibition is considered when:
A. The inhibitor structure is similar to substrate
one
B. The product of reaction is the allosteric
inhibitor for enzyme
C. There is the intermediate metabolite
formation from the inhibitor which tightly binds
to the active centre of enzyme to block it ---------D. The end-product of reaction binds to the
structure of the substrate to give non-soluble
complex
E. The end-product of reaction is not removed
from the environment
Choose the method that is more often used for the
determination of inhibitor type:
A. Nuclear magnetic resonance method
B. Michaelis-Menten graphical method
C. Briggs-Haldane graphical method
D. Lineweaver-Burk graphical method-----------E. Eadie-Hofstee graphical method
Find out the irreversible type of enzyme inhibition:
A. Competitive
B. Noncompetitive
C. Uncompetitive
D. Allosteric
E. Suicide --------------Point out the activator used for the determination of
amylase activity:
A. CuSO4

B. NaCl --------------C. H3PO4


D. ATP
E. Ca2+
The majority of key enzymes contain the allosteric
centre. Specify a role of this centre:
A. It attaches the substrate ---------B. It attaches the regulatory factor
C. It changes the structure of the substrate
D. It promotes the dissociation of a coenzyme
E. It blocks the active centre
Glycogen phosphorylase b is transformed to the active
form a by the action of special kinase with the use of
ATP as donor of phosphate group. Find out, please, the
type of enzyme activation:
A. Limited proteolysis
B. Covalent modification---------------------C. Activation by Ca2+
D. The change of pH medium
E. Competitive inhibition
Sulfonamides are used as drugs to protect our organism
from some bacteria. Enzyme in bacterial cell producing
folic acid from para-aminobenzoate is inhibited by this
group of drugs. Choose the type of inhibition for
Sulfonamides:
A. Competitive ----------------B. Noncompetitive
C. Uncompetitive
D. Allosteric
E. Suicide
Name, please, the inhibitor for salivary amylase:
A. Copper sulfate ----------------B. Sodium chloride
C. Potassium cyanide
D. Alanine
E. Hydrogen peroxide
Name the kinetic index that is changed under the
influence of competitive inhibitor on enzyme:
A. Michaelis constant
B. The initial velocity of enzymatic reaction ----C. The maximal velocity of enzymatic reaction
D. The dissociation constant of enzymesubstrate complex (ES)
E. The rate constant for the formation of ES
Find out, please, the factor used to change the charge of
functional groups both in the active centre of enzyme
and in the substrate molecule:
The temperature of environment
The pH of environment -----------------The addition of competitive inhibitor to the
environment
The addition of activator to the environment
All the factors proposed may be in need
The affinity of enzyme molecule to substrate one may be
estimated using the value of:
pH of the environment
The temperature
Km for the enzyme ----------------Vmax for reaction duration
The initial velocity of the reaction

Name, please, the equation for V-[S] dependence for the


moment of complete saturation of all the active centers
of enzyme molecules:
V=Vmax[S]/Km+[S] ---------V= kVmax
V=Vmax/Km+[S]
V=Vmax
V=k [S]
Cholesterol synthesis is regulated by feed-back
mechanism. Name the allosteric inhibitor of key enzyme
for this synthesis:
ATP
Cholesterol -------------Glucose
ADP
NADPH
Salivary amylase activity may be decreased by:
The change of pH from 6,8 to the value 5,5
The decrease of temperature from 38C to 25C
The addition of copper sulfate
The increase of temperature from 38C to 65C
All the changes described may be in need -------------Name, please, the factors that must be in constant levels
during the investigation of enzyme concentration
influence the velocity of enzymatic reaction:
pH of the environment
The temperature
Substrate concentration
Activator concentration
All the factors proposed must be in constant levels ------Lesson 6 Module 1
Point out the activator, used for the determination of
urine amylase activity under Volgemut's method:
A. CuSO4
B. NaCl ---------C. H3PO4
D. ATP
E. Ca2+
Patient's amylase activity in the urine excesses the
normal values in ten times as much. Point out the
possible diagnosis:
A. Viral hepatitis
B. Diabetes mellitus
C. Sharp pancreatitis --------------D. Influenza
E. Angina

Find out the term for unit of enzyme activity that is


estimated as the number of molecules of a substrate
catalyzed upon in a period 1 second by a single enzyme
molecule:
A. Total activity
B. Specific activity
C. Turnover number
D. Katal ---------------E. The Unit of an enzyme activity
Find out the substrate used for amylase activity
determination in the urine of patient:

A. Glucose
B. Pyruvate
C. Maltose
D. Glycogen
E. Starch --------------Find out the method for separation of isozymes to
determine their content in the blood serum of patient:
A. Dialysis
B. Electrophoresis --------------C. Spectrophotometry
D. Gel chromotography
E. Salting-out
There is the treatment of patients with achlorhydria
(the absence of free hydrochloric acid in the gastric
juice of patient) by enzyme as a drug. Name it:
A. Rennin
B. Pyruvate
C. Pepsin ------------------D. Trypsin
E. Chymotrypsin
Choose the enzyme used as diagnostic reagent for
glucose content determination in the blood:
A. Glucose-6-phosphatase
B. Pyruvate kinase
C. Maltase
D. Glucose oxidase -------------E. Amylase
A lot of factors must be taken into account to promote
methodic requirements for the determination of the
enzyme activity in biological fluids. Choose, please, the
most important from them:
A. pH of the environment
B. Temperature of the environment
C. Substrate concentration
D. Enzyme concentration
E. All the positions placed above --------------The determination of Lactate dehydrogenase (LDH)
isozymes content showed the increase of LDH 4 and
LDH5 fractions in the patient's blood plasma. Point out
the presumable diagnosis:
A. Viral hepatitis
B. Skeletal muscle dystrophy ----------------C. Diabetes mellitus
D. Myocardial infarction
E. Acute pancreatitis
Name, please, the reagent that is added to urine of
patient to increase the activity of amylase:
A. Sodium phosphate
B. Sodium chloride -----------------C. Copper sulfate
D. Glucose
E. Choline
Name the unit of the enzyme activity, if the reaction is
carried out by the quantity of the enzyme at a rate of 1
mol of the substrate conversion per second:
A. Standard international unit
B. Katal ------------------C. Specific activity
D. Turnover number
E. Conditional unit of activity

Patient's amylase activity in the urine equals 16 units.


Point out the possible state for this patient:
A. There is viral hepatitis in patient
B. There is diabetes mellitus in patient
C. There is the sharp pancreatitis in patient
D. The patient is apparently healthy -----------E. There is Angina in patient

change the pH of the incubation phase. Try to give the


name of this product:
A. Choline
B. Serum Albumin
C. Acetyl choline
D. Phosphatidyl choline
E. Acetic acid -------------------

Find out the term for unit of enzyme activity that is


estimated as the number of molecules of a substrate
acted upon in a period 1 second by a single enzyme
molecule:
A. Total activity
B. Special activity
C. Turnover number
D. Katal ------------------E. Unit of an enzyme activity

Genetic disorder associated with enzymatic pathology


may be caused by the:
A. Deficiency of the non-protein part of enzyme,
only
B. Disorder in the regulation of the transcription
of mRNA for enzyme synthesis --------------C. The damage of the feed-back mechanism of
enzyme regulation
D. Super-activation of inducer synthesis that is
used to stimulate transcription of mRNA for
enzyme
E. All the reasons described are right

Pancreatine is proposed as the drug to promote the


normal digestion of proteins in the small intestine of
patients with chronic pancreatitis. Choose the enzyme
that is the component of this drug and is possible to
destroy protein structure:
A. Amylase
B. Lipase
C. Pepsin
D. Trypsin ----------------E. Maltase
Competitive inhibitors of enzymes may be used as
drugs. Try to find out the medicine that is used to
decrease the rate of folic acid synthesis from paraaminobenzoic acid in microorganisms which can cause
the inflammation state of tissues in humans:
A. Antimycin A
B. Pancreatine
C. Phenobarbital
D. Sulfonamide ------------------------E. Phosphogluconate
Enzyme deficiency in patient usually is discussed as
severe form of pathology because:
A. It is genetic disorder that is difficult to treat -B. It is secondary reason of pathology
C. It is difficult to determine it
D. It cannot be prevented in the prenatal period
of organism development
E. It can be discovered in adults, only
We cannot use the enzymes as medicines for oral
administration because oral dose of enzyme:
A. Causes the allergic reactions in human
organism
B. Stimulates the production of albumins by the
liver
C. Can lead to the cleavage of blood plasma
proteins
D. Is digested in gastrointestinal tract -----------E. Changes the acid-alkaline balance in the
blood
The decrease of Choline esterase activity in the blood
serum of patient is the signal to care for the function of
one organ mainly. Name it:
A. Liver -----------------B. Spleen
C. Brain
D. Kidney
E. Pancreas gland
The principle of the method for Choline esterase
activity determination is based on the ability of the
product for the reaction catalyzed by this enzyme to

Find out, please, the value for amylase activity in the


urine (Volgemut`s method) corresponding the
pathological state - acute pancreatitis:
16 units
2 units
160 units -----------------32 units
8 units
Find out the type of Lactate dehydrogenase (LDH)
isozymes whose activity is in high level in the blood
plasma of patients at myocardium infarction:
LDH4 and LDH5
LDH3, only
LDH2 and LDH3
LDH1, only -----------------LDH5, only
The Acidic Phosphatase activity is determined in the
blood serum of patients with this pathologic state. Name
it:
Atherosclerosis of blood vessels
Myocardium infarction
Prostatitis ------------------Acute pancreatitis
Renal insufficiency
Point out the proteolytic enzyme of the blood that helps
to solvate the fibrin clot:
A. Plasminogen
B. Lysokinase
C. Plasmin ------------------D. Antifibrinolysinogen
E. Thromboplastin
Profuse foam appeared when dentist put hydrogen
peroxide on the mucous of the oral cavity.
What
enzyme caused such activity?
Catalase*
Cholinesterase
Acetyltransferase
Glucose-6-phosphatdehydrogenase
Methemoglobinreductase
Twelve hours after an accute attack of retrosternal pain
a patient presented a jump of aspartate

10

aminotransferase activity in blood serum. What


pathology is this deviation typical for?
Myocardium infarction *
Viral hepatitis
Collagenosis
Diabetes mellitus
Diabetes insipidus

Vitamin B1 (coenzyme TPP) is necessary for only one


dehydrogenase function in Krebs Cycle. Point out it:
A. Malate dehydrogenase
B. -Ketoglutarate dehydrogenase ----------C. Isocitrate dehydrogenase
D. Succinate dehydrogenase
E. Lactate dehydrogenase

Marked increase of activity of -forms of CPK


(creatine phosphokinase) and LDH-1 were revealed on
the examination of the patient's blood. What is the most
likely pathology?
Miocardial infarction*
Hepatitis
Rheumatism
Pancreatitis
Cholecystitis

Two reactions of Krebs Cycle are named as oxidative


decarboxylation. Point out the enzyme for this type of
reaction:
A. Citrate synthase
B. cis-Aconitate hydratase
C. Isocitrate dehydrogenase ------------------D. Succinate dehydrogenase
E. Succinyl~SCoA synthase

A patient presents high activity of LDH1, LDH2,


aspartate aminotransferase, creatine phosphokinase. In
what organ (organs) is the development of a
pathological process the most probable?
In the heart muscle (initial stage of myocardium
infarction)*
In skeletal muscles (dystrophy, atrophy)
In kidneys and adrenals
In connective tissue
In liver and kidneys
During metabolic process active forms of the oxygen
including superoxide anion radical are formed in the
human body. With help of what enzyme is this anion
inactivated?
Superoxide dismutase*
Catalase
Peroxidase
Glutathione peroxidase
Glutathione reductase
6 hours after the myocardial infarction a patient was
found to have elevated level of lactate dehydrogenase in
blood. What isozyme should be expected in this case?
A. LDH4
B. LDH1*
C. LDH5
D. LDH3
E. LDH2

Lesson 7, Module 1
Nucleoside triphosphate is formed in Krebs Cycle.
Point out its abbreviation:
A. ATP
B. CTP
C. GTP ---------------D. UTP
E. TTP
Only one dehydrogenase of Krebs Cycle has the nonprotein part FAD. Name it:
A. Isocitrate dehydrogenase
B. -Ketoglutarate dehydrogenase
C. Malate dehydrogenase
D. Succinate dehydrogenase -------------E. Pyruvate dehydrogenase

Name, please, the class of organic compound usually


used as energy source for anabolic pathways in humans:
A. Monosaccharides
B. Alcohols
C. Carboxylic acids
D. Nucleosides
E. Nucleoside triphosphates ------------Name, please, the process that is considered as the
second phase of catabolic pathways in humans:
A. Gluconeogenesis
B. Glycolysis -------------C. Urea cycle
D. Krebs cycle
E. Proteolysis
Choose, please, the transformation of intermediate
metabolites of Krebs cycle required the function of
multienzyme complex:
A. Citratecis-aconitate
B. Malate oxaloacetate
C. Isocitrate alpha-ketoglutarate
D. Alpha-ketoglutarate succinyl-CoA ---------E. Fumarate malate
Name, please, the conversion of Krebs cycle regulated
by inhibitor malonic acid
A. Succinyl-CoAsuccinate
B. Malate oxaloacetate
C. Isocitrate alpha-ketoglutarate
D. Succinatefumarate -------------E. Fumarate malate
Find out, please, the enzyme whose activity is inhibited
under the accumulation of ATP in the matrix of
mitochondria:
A. Aconitase
B. Malate dehydrogenase
C. Citrate synthase -------------D. Alpha-ketoglutarate dehydrogenase
E. Fumarase
How many stages are considered in catabolic pathway
for glucose up to the terminal products (carbon dioxide
and water):
A. Two
B. Three ------------C. One
D. Four
E. Five

11

Name, please, the key metabolite that may be formed in


catabolic pathway both for glucose and palmitic acid in
aerobic condition, only:
A. Pyruvate
B. Oxaloacetate
C. Acetyl-CoA ----------------D. Lactate
E. Malate
Name, please, the process that is anabolic pathway in
human organism:
A. Gluconeogenesis ---------------B. Glycolysis
C. Fatty Acid Oxidation
D. Krebs cycle
E. All the proposed
Name, please, the initial substrates for Krebs cycle (first
reaction):
A. Pyruvate and oxaloacetate
B. Oxaloacetate, only
C. Acetyl-CoA and oxaloacetate --------------D. Lactate and acetyl-CoA
E. Citric acid, only
Mg2+ and Mn2+ are in need for the function of one
enzyme from this register, only. Point out it:
A. Succinate dehydrogenase
B. Malate dehydrogenase
C. Isocitrate dehydrogenase -----------D. Aconitase
E. Fumarase
Find out the competitive inhibitor for succinate
dehydrogenase:
Malonic acid -------------Malic Acid
Fumaric acid
Citric Acid
Magnesium ion
Krebs cycle does not occur in:
Skeletal Muscle
Heart
RBC ------------Liver
All the above
Fluroacetate inhibits:
Citrate synthetase
Cis-Aconitase -----------------Succinate dehydrogenase
Alpha-ketoglutarate dehydrogenase
All these metabolic pathways or processes take place
inside the mitochondria except:
Glycolysis
Krebs cycle
Urea cycle
Oxidative phosphorylation
Fatty acid -oxidation ---------------Number of NADH molecules produced in Citric Acid
Cycle is:
2
3 ---------------------4
5
6

Name substances which are really terminal products for


catabolic pathways and for human organism:
Uric acid and Urea ------------------Carbon dioxide and Water
ATP and Carbon dioxide
A mino acids and Keto-acids
Bilirubin and Urea
Exogenous substances may be involved in catabolic
pathways to be used as energy sources for humans
EXCEPT:
Vitamins
Monosaccharides
Amino acids
Fatty acids
Alcohols ----------------------The accumulation of NADH in the matrix of
mitochondria is the signal to inhibit:
Citrate lyase
Cis-Aconitase
Isocitrate dehydrogenase ----------------Fumarase
Malate dehydrogenase
Amphybolic process must include intermediate
metabolites which are involved in both anabolic and
catabolic pathways of a cell. Choose those one:
Glycolysis
Hexose Monophosphate Shunt
Citric Acid Cycle ---------------Malate-aspartate shuttle system
All the proposed
Energy production is due to catabolic pathways only.
Name those one:
Gluconeogenesis
Citric Acid Cycle --------------Fatty Acid Elongation
Hexose Monophosphate Shunt
Glycogenesis
Propose the correct continuation of the phrase: Citric
Acid Cycle is:
The main producer of reduced forms of coenzymes
Anabolic process
Placed in cytoplasm of a cell
The main producer of energy for erythrocytes
Tissue respiration phase III ----------------How many moles of high energy bond containing
compound
are
produced
due
to
substrate
phosphorylation in one round of Citric Acid Cycle:
Twelve ---------------Two
One
Three
Four
The rate limited step for Citric Acid Cycle duration is
the reaction catalyzed by:
Citrate synthase
Cis-Aconitase
Isocitrate dehydrogenase
Alpa-ketoglutarate dehydrogenase
Malate dehydrogenase

12

Oxidative decarboxylation reactions occur two times in


Citric Acid Cycle, but the mechanism of these reactions
is not the same. Choose the conversion that may be
named as oxidative decarboxylation and catalyzed by
multienzyme system:
Citrate is converted to Cis-Aconitate
Isocitrate is converted to Alpa-ketoglutarate
Alpa-ketoglutarate is converted to Succinyl-CoA --------Malate is converted to Oxaloacetate
Succinate is converted to Fumarate
Name the regulatory enzyme from Citric Acid Cycle
whose activity is stimulated by allosteric activator ADP
at condition of its accumulation in the matrix of
mitochondria:
Citrate synthase
Cis-Aconitase
Isocitrate dehydrogenase ---------------------Alpa-ketoglutarate dehydrogenase
Succinate dehydrogenase
Krebs Cycle is the stage of catabolic ways in the
organism. Point, please, the number of stage, which is
corresponded to Krebs Cycle:
1
2
4
3 ---------------------------5
Krebs cycle is regulated by the ATP/ADP ratio in
aerobic cell. Point out the value of this ratio that causes
the stimulation of Krebs Cycle duration:
0.5 -------------------1
3
2.5
5
Isocitrate was used as an oxidized substrate in the
experiment with isolated mitochondria. Specify the
substance that can inhibit the isocitrate oxidative
decarboxylation:
ADP
Glucose
ATP -------------------Citrate
cAMP
Name, please, the enzyme from Krebs cycle catalyzing
the substrate phosphorylation:
. Succinyl-CoA thiokinase ---------------------. Fumarase
. Isocitrate dehydrogenase
D. Malate dehydrogenase
. Citrate synthase
The increase of one substrate concentration occurs the
mitochondrial matrix during the inhibition of Citrate
synthetase in the Krebs Cycle. Find out this substrate:
. Glucose
. Acetyl ~ SCoA -------------------------------. -Ketoglutarate
D. Malate
. Fumarate

Lesson 8, Module 1
Name, please, the inhibitor for complex IV of electron
transport chain:
Carbon dioxide
Potassium chloride
Hydrogen peroxide
Hydrogen sulfide ---------------Oxygen
Rotenone (the inhibitor of the first complex of the
electron transport chain) changes the P/O ratio for
substrates that are oxidized in Krebs Cycle. Choose the
value of P/O at the presence of this inhibitor per 1 mole
of malate that is oxidized:
. <1
. 0 ------------------------------. <3
D. <4
. <2
The isocitrate is converted into -ketoglutarate in the
Krebs cycle. Choose the substance that can low the P/O
ratio for this reaction:
Citrate
ADP
Isocitrate dehydrogenase
Antimycin A
NAD+
Find out, please, the inhibitor for the complex III of
electron transport chain in the inner membrane of
mitochondria:
Antimycin A ------------Rotenone
ATP
Barbituric acid
NADH
Catalase activity is very important for cells where the
accumulation of one toxic compound may be. Name,
please, this compound:
Carbon monoxide
Potassium cyanide
Nitric acid
Carbon dioxide
Hydrogen peroxide -------------------------The antibiotic oligomycin has been recently used in
tuberculosis treatment. Point out the process in
tuberculosis bacillus that is inhibited by this drug:
. Anaerobic glycolysis
. Translation
. Oxidative phosphorylation ------------------------------D. The active transport of substances across membranes
. Phagocytosis
Point out the location of the processes which take part
in tissue respiration the most intensively:
Lysosome
Mitochondria ------------------Cytoplasm
EPR
Golgi complex
Name, please, the energy effect per one Krebs cycle due
to oxidative phosphorylation:
8 ATP
12 ATP -----------------------1 GTP
11 ATP

13

9 ATP
Name, please, the name of enzyme catalyzing the
coupling of proton gradient with the synthesis of ATP
in mitochondria:
. Monoaminooxidase
. ATP/ADP-translocase
. ATP synthetase ---------------------------D. Lipase
. Pyruvate dehydrogenase
Find out, please, the name of the third stage of tissue
respiration:
. Oxidative phosphorylation
. Electron transport chain function ------------------. Aerobic glycolysis
D. The active transport of substances across membranes
. Translation
The electrochemical potential (H+) formation occurs
on the inner membrane of mitochondria during the
active work of the electron transport chain. Point the
substance that can reduce the H+ value:
. Succinate
. Isocitric acid
. 2,4-dinitrophenol --------------------D. Citric acid
. Glucose
Protons gradient in the inner membrane of
mitochondria may be increased under condition of
incorporation of high doses of one vitamin in the body.
Find out it:
Ascorbic acid
Folic acid
Cobalamin
Pantothenic acid
Retinoic acid
The tissue respiration is inhibited after coal gas
poisoning. Point the electron transport chain enzyme
whose activity abruptly reduces in these conditions:
. Succinate dehydrogenase
. NADH-dehydrogenase
. Cytochrome b1
D. Cytochrome c
. Cytochrome aa3 -----------------------------Name, please, the inhibitor for complex IV of
respiratory chain:
Carbon dioxide /x
Potassium chloride
Hydrogen peroxide
Carbon monoxide
Oxygen
Name, please, the substrate for Cytochrome oxidase
(reduced form):
. Succinyl-CoA
. Cytochrome C
. Isocitrate
D. Molecular oxygen ---------------------------. Water molecule
Find out the property of uncoupler that helps it to
penetrate across the inner membrane of mitochondria:
. High affinity to molecular oxygen
. High solubility in water
. Very small shape of uncoupler molecule

D. High solubility in lipids -----------------. All the properties proposed


Choose, please, the number of complexes in the long
respiratory chain:
A. Two
B. Three ---------------C. One
D. Four
E. Five
The complex I of electron transport chain may be
inhibited by barbituric acid. Find out, please, the
composition of this complex:
A. NAD and Fe-S-proteins
B. FAD and Fe-S-proteins
C. FMN and Fe-S-proteins ---------------D. NADH-dehydrogenase and Fe-S-proteins
E. Succinate dehydrogenase and Fe-S-proteins
Find out the enzyme of electron transport chain that is
transmembrane protein and may be considered as a
pump for protons from the matrix to the intra
membrane space:
A. Succinate dehydrogenase
B. Cytochrome C oxidase ----------------C. Cytochrome b, only
D. Cytochrome C
E. CoQ
Name, please, the factor that can block tissue
respiration completely:
A. Oligomycin
B. Potassium cyanide --------------------C. Phenobarbital
D. Malonic acid
E. Valinomycin
Name, please, the P/O ratio per one mole of pyruvate
involved in oxidative decarboxylation if 1 mole of
NADH is formed in this reaction:
A. 2
B. 3 ---------------C. 0
D. 4
E. 5
Choose, please, the P/O ratio per one mole of pyruvate
involved in oxidative decarboxylation at the presence of
phenobarbital in high levels in the cell:
A. 2
B. 3
C. 0 --------------D. 4
E. 5
Choose, please, the P/O ratio per one mole of pyruvate
involved in oxidative decarboxylation at the
accumulation of 2,4-dinitrophenol in the cell:
A. 2
B. 3
C. 0 ------------------D. 4
E. 5
Choose the enzyme of Krebs cycle that is considered as
the part of Complex II of electron transport chain:
A. Succinate dehydrogenase --------------B. Cytochrome oxidase
C. Cytochrome b, only
D. NaDH-dehydrogenase
E. CoQ

14

Name the component of electron transport chain that is


not related to proteins but is also acceptor of protons
and electrons:
A. Cytochrome c
B. Cytochrome aa3 -------------------------C. Cytochrome b
D. Cytochrome P450
E. CoQ
Name, please, the acceptor of electrons from NADHdehydrogenase in the electron transport chain:
A. Cytochrome c
B. Cytochrome oxidase
C. Cytochrome b
D. Fe-S-proteins
E. CoQ ----------------Name, please, the enzyme system promoting the
ADP/ATP maintenance both in the matrix of
mitochondria and in cytoplasm of the cell:
A. Succinate dehydrogenase
B. Cytochrome oxidase
C. ATP synthetase
D. NADH-dehydrogenase
E. ATP/ADP translocase ----------------Name, please the coefficient that can show the quantity
of ATP molecules formed per one mole of oxidized
organic compound due to oxidative phosphorylation:
A. Respiratory control
B. P/O ratio ----------------C. H+
D. Protons gradient
E. ADP/ATP ratio
Choose, please, the value for ATP/ADP ratio at the
moment when the tissue respiration has been just
inhibited in the cell:
A. ATP/ADP>1,5 -------------------B. ATP/ADP=0.8
C. ATP/ADP=0
D. ATP/ADP<1
E. ATP/ADP=1
Choose, please, the P/O ratio per one mole of isocitrate
involved in oxidative decarboxylation at the
accumulation of rotenone in the cell:
A. P/O<2
B. P/O<3
C. 0 -----------------D. 3
E. 2
Which of the following inhibit complex IV of
respiratory chain?
A. Amobarbital
B. Antipyrin
C. Secobarbitone
D. Cyanide -------------------------E. Rotenone

Acyl-CoA dehydrogenase involved in -oxidation of any


Fatty Acid, has the prosthetic group FAD+ and is placed
in the inner membrane of mitochondria of cells where
this process is in duration. How many moles of ATP may
be synthesized due to oxidative phosphorylation per 1
mole of acyl-CoA involved in Acyl-CoA dehydrogenase
reaction?
One --------------------------Two
Three
Four
Five

Antimycin A is discussed in scientific literature as the


antibiotic whose pharmacological effects are based on
its ability to inhibit complex III of electron transport
chain of the inner membrane of mitochondria. Choose
the value for P/O ratio of succinate oxidation in Krebs
Cycle at the presence of antimycin A in mitochondria:
0 --------------------------2
3
Less then 3
Less then 2
ATP/ADP ratio equals 0.5 in the matrix of suspended
mitochondria. What enzyme systems must be stimulated
in the matrix and inner membrane to increase this value
up to 1?:
Citric Acid Cycle enzymes
Electron transport chain enzymes
ATP-synthetase
ATP/ADP-translocase
All the proposed --------------------------Carbon monoxide is discussed as the suppressor of
tissue respiration because of its ability to be linked to:
Hemoglobin
Cytochrome C oxidase -------------------Myoglobin
All the positions are right
What type of tissue is specialized to carry out the
oxidation uncoupled from phosphorylation?:
White adipose tissue
Erythrocytes of blood
Brown adipose tissue
Skeletal muscular tissue ----------------------Myocardium tissue

Cytochromes are:
Pyridine nucleotides
Metal containing flavoproteins
Peroxidases
Iron-porphyrin proteins-------------Hemoglobin derivatives
Uncouplers of oxidative phosphorylation can be:
Thermogenins
High levels of serum bilirubin
High levels of thyroid hormones -----------------High levels of free fatty acids
All the proproposed

15

Point out the endopeptidase that is


produced by pancreas and is activated by
trypsin:
Proelastase*
Renin
Pepsinogen
Gastricsin
Alpha-Amylase

Lesson 10 module 1
The only sugar absorbed by intestine against
a concentration gradient is:
Glucose*
Mannose
Erythrose
Xylose
Fructose
The bile acids participate in:
The activation of trypsin
The lipids emulsification*
The cholesterol synthesis
The ketone bodies synthesis
The protein transport activation
Point out the class of enzymes that catalyze
the digestion of proteins in gastro-intestinal
tract:
Transferases
Lyases
Hydrolases*
Oxidoreductases
Ligases
Point out the group of peptidases which
trypsin is related to:
Amino peptidase
Exopeptidase
Endopeptidase*
Dipeptidase
Carboxypeptidase
Point out the couple of amino acids
participating in the formation of peptide
bond that is cleaved by trypsin:
Arginine, lysine*
Leucine, valine
Glycine, Glutamine
Alanine, valine
Isoleucine, alanine

Find out the values for total acidity of


gastric
juice
associated
with
hypochlorhydria in patient:
40 mmol/L
50 mmol/L
60 mmol/L
20 mmol/L*
55 mmol/L
Name the enzyme whose function is
associated with digestion of polysaccharides
in the small intestine:
Elastase
Rennin
Pepsinogen
Maltase
Alpha-Amylase*
A 30-year-old woman was diagnosed with
insufficiency of exocrine function of
pancreas. Hydrolysis of what nutrients will
be disturbed?
Proteins, fats, carbohydrates*
Fats, carbohydrates
Proteins, carbohydrates
Proteins, fats
Proteins
Examination of a man who hadnt been
consuming fats but had been getting enough
carbohydrates and proteins for long time
revealed dermatitis, poor wound healing,
vision impairment. What is the probable
cause of metabolic disorder?
Lack of vitamins PP, H
Lack of oleic acid
Lack of linoleic acid, vitamins A,
D, E, K*
Lack of palmitic acid
Low caloric value of diet

16

After intake of rich food a patient suffered


from nausea and sluggishness, and then
signs of
steatorrhea appeared. Blood
cholesterol concentration for this patient was
9.2 mmole/l. That condition was caused by
lack of:
Chylomicrons
Fatty acids
Phospholipids
Bile acids*
Triglycerides
A patient complains of frequent diarrheas,
especially after consumption of fattening
food and of the body weight loss.
Laboratory
examination
revealed
steatorrhea; hypocholic feces. What may be
the cause of this condition?
Mucous membrane inflammation
of small intestine
Obturation of biliary tracts*
Lack of pancreatic phospholipase
Lack of pancreatic lipase
Unbalanced diet
Choose the enzyme that takes part in the
digestion of carbohydrates in adults:
Oligo-1,6-glycosidase
Amylo-1,6-glycosidase
Salivary amylase
There's
no
any
enzyme
for
carbohydrates digestion
All the enzymes placed *
The vegetable oils are the obligatory
components of man ration as they contain
some essential fatty acids. Choose them:
Stearic and palmitic acids
Acetic and butyric acids
Palmitooleic and oleic acids
Linoleic and -linolenic acids*
Citric and fumaric acids
The stone of a general bile duct stopped the
entrance of bile to the intestine of the
patient. Specify the process, whose
infringement will be observed:
Protein digestion
Carbohydrates absorption
Fats digestion*
Protein digestion
Carbohydrates digestion

Point out the amino acids whose peptide


bonds are hydrolyzed by pepsin and
chymotrypsin:
Diamino acids
Aromatic amino acids*
Dicarboxylic acids
Alcoholic acids
Sulfur-containing amino acids
Some carbohydrates can't be digested in the
intestine. Point out one of them:
Starch
Sucrose
Cellulose*
Glycogen
Maltose
Point out the activator
production in duodenum:
Hydrochloric acid*
Pepsin
Gastricsin
Trypsin
Chymotrypsin

of

secretin

This disorder may be genetic or acquired.


Affected infants are not able to tolerate milk,
which is normally their primary food. Point
out the name of disorder:
Lactose intolerance*
Galactose intolerance
Glucose intolerance
Fructosuria
Galactosemia
Lipase requires special conditions for its
activity. Choose a factor that isnt optimal
for this purpose:
Bile salts
pH=8.0
Colipase
pH=2.0*
Phospholipids
Achlorhydria state is observed in patient
with stomach problems. Propose probable
reason for the development of this state and
probable values for free HCL content in the
gastric juice of this patient:

17

Obstruction of esophagus; [free HCL]=20


mmole/L
Disturbed pepsinogen synthesis; [free
HCL]=10 mmole/L
Cancer of stomach; [free HCL]= 0 *
Colitis; [free HCL]=60 mmole/L
Extensive secretion of gastric juice; [free
HCL]=60 mmole/L
Trypsin is found in functions as activator for
some zymogens produced by pancreas.
Choose them:
Proelastase
Procarboxylase A
Procarboxylase B
Chymotrypsinogen
All that is placed *
Due to the blockage of the common bile
duct (which was radiographically
confirmed), the biliary flow to the
duodenum was stopped. We should expect
the impairment of:
Carbohydrate hydrolysis
Protein absorption
Secretion of hydrochloric acid
Salivation inhibition
Fat emulsification *
To prevent attacks of acute pancreatitis a
doctor prescribed the patient trasylol
(contrycal, gordox), which is an inhibitor of:
Trypsin*
Carboxypeptidase
Elastase
Chymotripsin
Gastricsin
A patient has normally coloured stool
including a large amount of free fatty acids.
The reason for this is a disturbance of the
following process:
Fat hydrolysis
Fat absorption *
Biliary excretion
Lipase secretion
Choleresis
Lesson 11 module 1
What enzyme catalyzes the glucose-6phosphate formation from glucose in the
liver and is not inhibited by excess level of
glucose-6-phosphate:

Hexokinase
Glucokinase*
Pyruvate kinase
Glucose-6-phosphatase
Phosphoglucomutase
Choose the condition in human organism
which can cause the beginning of
gluconeogenesis in the liver:
Hyperglycemia
Hypoglycemia*
The decrease of diuresis
The hypoxia of liver tissue
The bile ducts obstruction
Name the factors which are important to
regulate the anaerobic glycolysis duration:
ATP/ADP ratio in a cell
NADH/NAD+ ratio in a cell
Fructose-2.6-biphosphate level
Oxygen level in tissue
All the factors mentioned*
Choose the enzyme for the reaction of
glucose
formation
due
to
dephosphorylation:
Glucokinase
Phosphofructokinase
Glucose-6-phosphatase*
Aldolase
Find out the location of
phosphatase in human tissues:
Gonads, only
Liver, kidney*
Liver, only
Skeletal muscular tissue
Myocardium

glucose-6-

Name the energy effect of anaerobic


glycolysis per 1 mole of glucose
incorporated into the process:
2 ATP*
5 ATP
8 ATP
10 ATP
3 ATP
A 7-year-old girl has signs of anemia.
Laboratory examination revealed pyruvate
kinase deficiency in the erythrocytes. What

18

process disturbance plays the main role in


anemia development?
Tissue respiration
Oxidative phosphorylation
Peroxide decomposition
Amino acids desamination
Anaerobic glycolysis*
The gluconeogenesis is activated in the liver
after intensive physical trainings .What
substance is utilized in gluconeogenesis first
of all in this case:
Glucose
Glutamate
Alanine
Lactate*
Pyruvate
Which one of the following enzymes
catalyzes phosphorylation with the use of
inorganic phosphate?
Glyceraldehyde-3-phosphate
dehydrogenase*
Hexokinase
Pyruvate kinase
Phosphofructokinase
Phosphoglycerate kinase
Which of the following compounds is an
allosteric activator that enhances activity of
phosphofructokinase I?
Glucose
Glucose-6-phosphate
Citric acid
Adenosine monophosphate (AMP)*
Adenosine triphosphate (ATP)
A deficiency in the vitamin biotin leads to a
deficiency in the activity of certain enzyme
of gluconeogenesis. Poin out the enzyme:
Phosphoenolpyruvate carboxykinase
Pyruvate carboxylase*
Fructose-1.6-bisphosphatase
Phosphoglycerate kinase
Glucose 6-phosphatase
Choose the yields a net for 1 mole of
glucose oxidation by anaerobic glycolysis:
2 moles of lactate and 2 moles of ATP*
2 moles of lactate, 2 moles of NAD+, 2
moles of ATP

2 moles of pyruvate, 2 moles of NADH, 2


moles of ATP
2 moles of pyruvate and 2 moles of ATP
2 moles of lactate and 6 moles of ATP
Which one of the following enzymes is
common
to
both
glycolysis
and
gluconeogenesis?
Pyruvate kinase
Pyruvate carboxylase
Hexokinase
Phosphoglycerate kinase*
Fructose-1,6-bisphosphatase
Which compound does not contribute to the
net formation of glucose during
gluconeogenesis in man?
Oxaloacetate
Glycerol
Glyceraldehyde 3-phosphate
Lactate
Acetyl-CoA*
Which statement about gluconeogenesis is
correct?
The acetate group of acetyl-CoA is used for
the net synthesis of glucose.
It occurs primarily in skeletal muscle.
Lactate and alanine can both serve as
substrates *
Phosphofructokinase I is the rate-limiting
enzyme of gluconeogenesis.
ATP is not required for the process
Choose
the
key
enzymes
of
gluconeogenesis:
Glucokinase, phosphotriose isomerase,
lactate dehydrogenase
Hexokinase, phosphofructokinase, pyruvate
kinase
Citrate synthase, isocitrate dehydrohenase,
-ketoglutarate dehydrogenase
Phosphohexose
isomerase,
phosphoglycerate kinase, aldolase I
Pyruvate carboxylase, phosphoenolpyruvate
carboxykinase, fructose-1,6-bisphosphatase*
The intensive muscular work increases
the velocity of the ATP utilization and
causes the activation of glycolysis. What
enzyme activity will limit its rate?
Aldolase
19

Glucose-6-phosphate isomerase
Phosphoglyceromutase
Phosphofructokinase*
Phosphoglycerate kinase
There is one organic compound in the list
below to inhibit phosphofructokinase I if
this compound is accumulated in
cytoplasm. Name it:
Glyceroaldehyde 3-phosphate
Succinate
ATP*
Fructose-6-phosphate
Pyruvate
Name, please, the liver enzyme needed for
lactate utilization in gluconeogenesis:
NAD-dependent Lactate Dehydrogenase*
NADH-dependent Lactate Dehydrogenase
Pyruvate dehydrogenase
Glucose-6phosphatase
Lactonase
The functions of gluconeogenesis are
described by all of the following statements
except:
It maintains blood sugar levels during
fasting
It is useful during strenuous exercises
It allows the use of acetyl-CoA for glucose
production*
It allows the use of amino acids for glucose
production
It maintains blood glucose level during
period of limited carbohydrate intake
Choose the key (regulatory) enzymes of
glycolysis:
Glucokinase, phosphotriose isomerase,
lactate dehydrogenase
Hexokinase, phosphofructokinase, pyruvate
kinase *
Citrate synthase, isocitrate dehydrohenase,
-ketoglutarate dehydrogenase
Phosphohexose
isomerase,
phosphoglycerate kinase, aldolase I
Fructose-1,6-bisphosphatase,
phosphoenolpyruvate
carboxykinase,
pyruvate carboxylase

Gluconeogenesis from lactate does not


require activity of:
Aldolase
Phosphofructokinase *
Glyceraldehyde 3-phosphate dehydrogenase
Triose phosphate isomerase
Phosphoglycerate kinase
The activity of pyruvate carboxylase is
dependent upon the positive allosteric
effector:
Succinate
AMP
Acetyl CoA*
Isocitrate
Citrate
What ions are the most important for
Glycolysis?:
Ca2+
Mg2+*
Na+
Zn2+
Co2+
Gluconeogenesis occurs in the liver and:
Heart
Kidney*
Skeletal muscle
Adipose tissue
Spleen
Diseases of the respiratory system and
circulatory disorders impair the transport of
oxygen, thus leading to hypoxia. Under
these conditions the energy metabolism is
carried out by anaerobic glycolysis. As a
result, the following substance is generated
and accumulated in blood:
Fumaric acid
Pyruvic acid
Glutamic acid
Citric acid
Lactic acid *
The genetic defect of pyruvate carboxylase
deficiency is the cause of delayed physical
and mental development and early death in
children. This defect is characterized by
lacticemia, lactaciduria, disorders of a
number of metabolic pathways. In particular,
the following processes are inhibited:
20

Citric acid cycle and gluconeogenesis *


Glycolysis and glycogenolysis
Pentose phosphate pathway and
glycolysis
Lipolysis and lipogenesis
Glycogenesis and glycogenolysis
Choose the energy requirement (sum total
of ATP; to think about GTP use as ATP
use) for glucose synthesis from 2 moles of
pyruvic acid:
2 ATP
6 ATP*
1 ATP
4 ATP
3 ATP
Lesson 12 module 1
A childs blood presents high content of
galactose, glucose concentration is low.
There are such presentations as cataract,
mental deficiency, adipose degeneration of
liver. What disease is it?
Diabetes mellitus
Steroid diabetes
Fructosemia
Lactosemia
Galactosemia*
A child has got galactosemia. Concentration
of glucose in blood has not considerably
changed. Deficiency of what enzyme caused
this illness?
Phosphoglucomutase
Amylo-l,6-glucosidase
Galactokinase
Galactose-1-phosphate uridyltransferase*
Hexokinase
Point out the multienzyme system used in
aerobic oxidation of glucose up to carbon
dioxide and water formation:
-Ketoglutarate dehydrogenase complex *
Glucose-6-Phosphate
dehydrogenase
complex
Glutamate dehydrogenase complex
Succinate dehydrogenase complex
Palmitate synthetase
Pyruvate concentration in the patients urine
is increased 10 times than the normal level.
Choose the Vitamin, the deficiency of which

in the organism can be the reason of this


change:
Vitamin C
Vitamin B1*
Vitamin A
Vitamin E
Vitamin B6
The aerobic oxidation of carbohydrates
includes the oxidative decarboxylation of
pyruvate. Specify the inhibitor (in high
concentration) for this reaction:
ADP
NAD+
FAD
Acetyl-SCoA *
NADP+
Name the second stage of aerobic glucose
oxidation in tissues:
Krebs Cycle
Oxidative decarboxylation of pyruvate*
Pyruvate formation
-Ketoglutarate formation
Acetyl-SCA carboxylation
Point out a substrate that is used as major
energy source for brain:
Glucose *
Fatty acid
Phospholipid
Ketone body
Alanine
Choose the INCORRECT statement
concerning functions of the pentose
phosphate pathway:
It is a source of NADPH for reductive
pathways
It is a source of ATP for reductive pathways
It is a source of pentoses for nucleic acids
synthesis
It is a route for the use of pentoses and for
their conversion to hexoses and trioses
It is a route for conversion of pentoses from
degradated nucleotides to intermediates of
glycolysis
NADPH is used for next application
EXCEPT:
Fatty acid biosynthesis
Cholesterol synthesis
21

Glucose synthesis*
Glutamate synthesis
Glutathione defense system function against
injury by reactive oxygen species
Which of the following compounds is
required by transketolase for maximal
activity?
Thiamine pyrophosphate*
Biotin
Coenzyme A
Dihydroxyacetone phosphate
Acetyl-CoA
A child with galactosemia is referred to an
opthalmologist for evaluation of developing
cataracts, probably caused by the
accumulation of galactose and its alcohol
galactitol. Choose the enzyme catalyzing the
transformation of galactose to galactitol:
Galactokinase
Aldose reductase*
Glucokinase
Galactose 1-phosphate uridyltransferase
Aldolase B
Which enzyme is present in the liver but not
in muscle?
Glucose 6-phosphatase*
Hexokinase
Lactate dehydrogenase
Pyruvate dehydrogenase
Glycogen phosphorylase
Biochemical test in thiamine deficiency may
be:
Cholesterol test
Histidine test
Erythrocyte transketolase test*
Ferric chloride test
Glucose test

Gluconeogenesis
Pentose phosphate pathway
Embden-Meyerhoff pathway
Reducing equivalent (NADH) transport into
mitochondria*
Alanine transport into mitochondria
Pyruvate dehydrogenase complex contains
all cofactors except one. Choose it:
Biotin *
NAD+
FAD
CoA
Lipoamide
Immediate products of pyruvate metabolism
(using one reaction only) are all except:
Acetyl-CoA
Alanine
Lactate
Oxaloacetate
2-Phosphoglycerate*

Point out the multienzyme system where


enzymes form a single structural-functional
complex:
Enzymes of Krebs cycle
Pyruvate dehydrogenase complex*
Glutamate dehydrogenase
Glucose-6-Phosphate dehydrogenase
Carbamoyl phosphate synthetase
Lesson 13
How glucocorticoids
influence
the
carbohydrate metabolism?
Stimulate the glycolysis from glucose
Stimulate the gluconeogenesis*
Stimulate the starch hydrolysis in the
small intestine
Inhibit the glycogen phosphorolysis
Stimulate the glycogenesis

Sites where Hexose monophosphate shunt


can occur include:
Liver
Lactating mammary gland
White Blood Cells (WBC)
Testes
All the proposed*

Which one of the following enzymes is


associated with glycogen synthesis?
A. Branching enzyme*
B. Glycogen phosphorylase
C. Phosphorylase kinase
D. Debranching enzyme
E. Glucose-6-phosphatase

Malate-aspartate shuttle system is required


for:

Choose the hormone whose secretion may


be damaged at diabetes mellitus in person:
22

A. Cortisol
B. Progesterone
C. Growth hormone
D. Insulin*
E. Epinephrine
Which of the following statements about
glycogen metabolism is true?
A. Cyclic AMPactivated protein kinase
stimulates glycogen synthase
B. Glycogen phosphorylase kinase is
activated by phosphorylation*
C. Glycogen phosphorylase b is inactivated
by phosphorylation
D. Epinephrine and glucagon can low
cAMP levels in cytoplasma thus stimulating
glycogenolysis
E. Glycogen synthesis is stimulated by
glucagon
Point out the main process maintaining the
blood glucose level during fasting first 24
hours:
A. Glycolysis
B. Hexose monophosphate shunt
C. Glycogenolysis in the muscles
D. Glycogenolysis in the liver*
E. Gluconeogenesis in the muscles
A child is languid, apathetic. Liver is
enlarged and liver biopsy revealed a
significant amount of glycogen. Glucose
concentration in the blood stream is below
normal. What is the cause of low glucose
levels:
A. Low {absent} activity of hexokinase
B. High activity of glycogen synthetase
C. Deficit of gene that is responsible for the
synthesis of glucose 1-phosphate uridine
transferase
D. High activity of glycogen phosphorylase
in liver
E. Low {absent} activity of glucose 6phosphatase*
A child has been diagnized for low blood
glucose (hypoglycemia), enlarged liver
(hepatomegaly), and excess fat deposition in
the cheeks. A liver biopsy reveals excess
glycogen in hepatocytes. There is
hypothetically Pompes desease. Deficiency

of which of the following enzymes might


explain this phenotype?
A. Glucose 6-phosphatase
B. Glucosyl -4:6 transferase (branching
enzyme)
C. Lysosomal alpha-1,4-glucosidase*
D. Alpha-1,4-galactosidase
E. Amylo
alpha-1,6
glucosidase
(debranching enzyme)
Point out the enzyme that breaks beta14
linkages in hyaluronic acid and other
glycosaminoglycans?
A. Cellulase
B. Glycogen phosphorylase
C. Lactase
D. Hyaluronidase *
E. Glycogen synthase
Point out the enzyme whose deficiency can
cause the Gierke's disease development:
A. Alpha-1,4-glycosidase
B. Amylo- 1,6 -glycosidase
C. Glycogen-branching enzyme
D. Glucose-6-phosphatase*
E. Glycogen phosphorylase
Which glycosaminoglycan is the most
abundant in cartilage?
A. Heparan sulfate
B. Keratan sulfate
C. Chondroitin sulfate *
D. Hyaluronic acid
E. Dermatan sulfate
After meals blood glucose enters liver cells
and is stored as glycogen, mainly. Which of
the following compounds is the donor of
new glucose residue in glycogen molecule?
A. UDP-glucose-1-phosphate
B. UDP-glucose*
C. UDP-glucose-6-phosphate
D. Glucose-6-phosphate
E. Glucose-4-phosphate
McArdles disease development causes
muscle cramps and muscle fatigue with
increased muscle glycogen in a patient.
Which of the following enzymes is
deficient?
A. Hepatic hexokinase
B. Muscle glycogen synthetase
23

C. Muscle glycogen phosphorylase*


D. Muscle hexokinase
E. Muscle debranching enzyme
Find out the pair of enzymes providing the
reciprocal control of carbohydrate metabolic
pathways:
A. Acetyl CoA carboxylase and pyruvate
kinase
B. Hexokinase and glucose 6-phosphatase
C. Phosphohexose isomerase ang
phosphotriose isomerase
D. Branching and debranching enzymes
E. Glycogen synthase and glycogen
phosphorylase*
Insulin decreases the rate of
gluconeogenesis by suppressing of one the
following enzymes. Point out this enzyme:
A. Hexokinase
B. Phosphofructokinase
C. Pyruvate kinase
D. Glycogen synthase
E. Pyruvate carboxylase*
A liver biopsy has been made for child with
hepatomegaly
and
mild
fasting
hypoglycemia. Hepatocytes contant shows
the accumulation of glycogen granules with
single glucose residues remaining at the
branch points near the periphery of the
granule. The most likely genetic disorder in
this child is associated with the defect in:
A. Glycogen phosphorylase
B. Glucosyl -4:6 transferase (branching
enzyme)
C. Phosphoglucomutase
D. Amylo alpha-1,6 glucosidase
(debranching enzyme)*
E. Lysosomal alpha-1,4 glucosidase
Insulin stimulates all the processes listed
below in adipocytes except:
A. Oxidative decarboxylation of pyruvate
B. Hexose monophosphate shunt
C. Transport of glucose through cell
membrane
D. Catabolism of glucose
E. Glycogenolysis*

The most important substances as a source


of blood glucose after 48 hours of starvation
are:
A. Muscle and liver glycogen
B. Liver glucoseaminoglycans
C. Fructose and galactose
D. Amino acids*
E. Fructose
What hormone decreases the glucose
concentration in the blood, if its value is
more than 6.8 mM/L:
A. Thyroxin
B. Testosterone
C. Glucagon
D. Adrenalin
E. Insulin*
Choose the substance that may be the
terminal product of glycogenolysis in
muscles:
A. Glycogen
B. Glucose
C. Pyruvate
D. Fructose
E. Glucose-6-phosphate*
Point out the key enzyme of glycogen
degradation in the liver:
. Fructose-1,6-bisphosphatase
B. Glycogen Phosphorylase*
C. Glyceraldehyde-3-phosphatase
D. Glucose-6-phosphatase
E. Glucose oxidase
How does adrenalin influence the glucose
level in the blood?
A. Increases, stimulating the glycogen
destruction*
B.
Decreases,
stimulating
the
gluconeogenesis
C. Does not influence
D. Decreases, inhibiting the glycogen
synthesis
. Decreases, inhibiting the glycolysis
Patient with diabetes mellitus experienced
loss of consciousness and convulsions after
an injection of insulin. What might be the
result of biochemical blood analysis for
concentration of glucose?
A. 3,3 mmole/L
B. 10 mmole/L
24

C. 8,0 mmole/L
D. 1, 5 mmole/L .*
E. 5,5 mmole/L
The investigation of the blood and urine
of patient with diabetes mellitus
confirmed the hyperglycemia and
glucosuria states. Point the available
value of glucose concentration in the
blood plasma of this patient:
A. 2, 54 mmol/l
B. 3, 88 mmol/l
C. 9, 32 mmol/l*
D. 6, 55 mmol/l
E. 4, 89 mmol/l
Which of the following metabolites is found
in
glycogenolysis,
glycolysis,
and
gluconeogenesis?
A. Galactose-1-phosphate
B. Glucose-6-phosphate*
C. Uridine diphosphoglucose
D. Fructose-6-phosphate
E. Uridine diphosphogalactose
Which of the following alterations would be
expected to occur after a substantial
breakfast?
A. Increased
activity
of
pyruvate
carboxylase
B. Decreased activity of glucokinase
C. Decreased rate of glycogenolysis*
D. Decreased rate of protein synthesis
E. Increased
activity
of
phosphoenolpyruvate carboxykinase
Blood glucose levels cannot be augmented by
mobilization of muscle glycogen due to lack
of:
A. Glucose-6-phosphate dehydrogenase
B. Aldolase
C. Glucose-6-phosphatase*
D. Glucokinase
E. Hexokinase
The tissue with the highest glycogen content
(mg/100g of tissue) is:
A. Liver*
B. Skeletal muscle
C. Kidney
D. Testes
E. Brain

Mucopolysaccharidoses are inherited storage diseases.


They are caused by:
A. An increased rate of synthesis of proteoglycans
B. The synthesis of polysaccharides with an altered structure
C. Defects in the degradation of proteoglycans
D. The synthesis of abnormally small amounts of protein
cores
E. All

the proposed options are correct*

Glycogen polysaccharide is synthesized


from the active form of glucose. The
immediate donor of glucose residues during
the glycogenesis is:
A. Glucose-3-phosphate
B. Glucose-6-phosphate
C. UDP-glucose*
D. Glucose-1-phosphate
E. ADP-glucose
Inherited disease, such as
mucopolysaccharidosis, is manifested in
metabolic disorders of connective tissue,
bone and joint pathologies. The sign of this
disease is the excessive urinary excretion of
the following substance:
A. Lipids
B. Amino acids
C. Urea
D. Glycosaminoglycans *
E. Glucose
A 50-year old patient with food poisoning is
on a drip of 10 % glucose solution. It does
not provide the body with necessary energy
only, but also performs the function of
detoxification by the production of a
metabolite that participates in the following
conjugation reaction:
A.
B.
C.
D.
E.

Glucuronidation *
Hydroxylation
Methylation
Sulfation
Glycosylation

What glycolytic enzyme is stimulated by


insulin:
Glucose-6-phosphatase
Hexokinase*
Aldolase
Adenylate kinase
Phosphoglycerate kinase
Lesson 14 Module 1
25

The lipids are transported by lipoproteins in


the blood. Specify the lipoproteins that are
formed in the small intestine wall after high
lipids intake:
A. HDL
B. Chylomicrons*
C. LDL
D. VLDL
E. IDL
Point out the lipoprotein class transporting
cholesterol from peripheral tissues to the
liver:
A. VLDL
B. LDL
C. HDL*
D. Chylomicrones
E. Cholesterol-albumin complex
Certain drugs -caffeine and theophyllineinhibit phosphodiesterase activity catalyzing
reaction cAMPAMP. How will free fatty
acid levels are altered in the blood after the
drugs injection:
A. Will be increased*
B. Will be decreased
C. Will be not altered
D. Will be increased simultaneously with
galactose
E. Will be decreased simultaneously with
glucose

B. Brain, pancreas gland


C. Kidneys, adrenal gland
D. Liver, Mammary gland*
E. Myocardium
Which one of the following properties is not
characteristic of LDL?
A. LDL is smaller than both VLDL and
chylomicrones
B. LDL contains more cholesteryl esters
than triacylglycerols
C. The major protein component of LDL is
apoB-48 *
D. LDL are more dense than chylomicrons
E. LDL have specific high affinity receptors
in most cells
Which statement is incorrect about
phosphatidylcholine (PC)?
A. PC may be synthesized from
phosphatidylserine or from diacylglycerol
and CDP-choline.
B.
PC
together
with
phosphatidylethanolamine
and
phosphatidylserine are major phospholipid
components of cell membranes.
C. PC participates in signal transmission via
activated phospholipase C*
D. PC is an important component of the
surface shell of plasma lipoproteins.
E. PC can be hydrolyzed by phospholipase
A2 into lysolecithin and a fatty acid.

The liver requires a mechanism for


producing phosphatidylcholine (PC) because
it exports significant amounts of PC in the
bile and as component of serum
lipoproteins. This mechanism includes three
methylation steps to produce PC from
phosphatidylethanolamine. What is the
methyl group donor for methylation:
A. S-adenosylmethionine (SAM)*
B. N-guanosylmethionine (NGM)
C. Cytidine diphosphate-choline (CDPcholine)
D. Uridine diphosphate-methionine (UDPmethionine)
E. Homosycteine

What substance is not a component of


lipoprotein:
A. Cholesterol
B. Triacylglycerol
C. Phospholipid
D. Apoprotein
E. Transferrin*

Choose the organs or tissues where


lipogenesis proceeds most intensively:
A. Skeletal Muscle

Name the condition in the liver cell causing


the triacylglycerol synthesis:
A. The decrease of ATP/ADP ratio

Choose the separation method


lipoproteins of the blood plasm:
A. Radioimmunal Assay
B. Extraction
C. Electrophoresis*
D. Salting-out
E. Photocolorimetry method

for

26

B. The accumulation of high fatty acids *


C. The accumulation of carbon dioxide
D. The ketone bodies accumulation
E. The stimulation of protein degradation
Which statement is a true for the comparison
of phospholipids (PL) and triacylglycerols
(TAG)?
A. Both molecules contain a phosphate
group.
B. TAG may be saturated or unsaturated,
but all fatty acids in PL structure are
saturated
C. Both TAG and PL are the primary
storage form for fats in our bodies.
D. PL contein a cyclic steroid ring, whilst
triglycerides maintain a straight-chain form.
E. PL molecules have a distinctly polar
'head' and a distinctly non-polar 'tail,' whilst
triglycerides are predominantly non-polar*
Point out the enzyme used for
lysophospholipid formation during lipolysis
of glycerophospholipid:
A. Diacylglycerol lipase
B. Monoacylglycerol lipase
C. Triacylglycerol lipase
D. Phospholipase A2*
E. Phospholipase D
Synthesis of phospholipids is disordered
under the liver fat infiltration. Indicate
which of the following substances can
enhance the process of methylation during
phospholipids synthesis?
A. Glucose
B. Citrate
C. Methionine*
D. Glycerol
E. Ascorbic acid
Vitamin F is recommended in order to
prevent the cholesterol deposition inside
the blood vessels at atherosclerosis. What
lipid is the component of this vitamin?
A. Linoleic acid*
B. Stearic acid
C. Sphingomyelin
D. Cholesterol
E. Cardiolipid

Hormone-sensitive triacylglycerol lipase is


activated in adipocytes under the emotional
stress. Choose the substance, whose
concentration is increased in adipose tissue
in this state:
A. cAMP*
B. cGMP
C. AMP
D. Diacylglycerol
E. Ca2+
Blood
plasma
of
patient
with
hyperlipoproteinemia type I remains milky
even after a long fast due to markedly
elevated and persistent chylomicrons. What
abnormality is possible in these patients?
A. Obstruction of the bile duct
B. Deficient pancreatic lipase
C. Defective synthesis of apoB-48
D. Deficient
lecithin
cholesterol
acyltransferase (LCAT)
E. Deficient lipoprotein lipase*
If choline moiety is replaced by
ethanolamine in glycerophospholipid the net
product is:
A. Cerebroside
B. Sphingomyelin
C. Cephalin*
D. Plasmalogen
E. Cardiolipin
Which is a lipotropic factor:
A. Glycerol
B. Insulin
C. Choline*
D. Carnitine
E. Alanine
Which of the following is an activator of
LCAT (Lecithin Cholesterol
Acyltransferase)
A. Apo B 100
B. Apo B 48
C. Apo E
D. Apo AI*
E. Apo CII
Free fatty acids are transported in circulation
via:
A. Albumins*
B. Globulins
27

C. Triacylglycerols
D. Chylomicrons
E. HDL
In man the serum lipoprotein fraction with
the highest free cholesterol content is:
A. Prealbumin
B. Beta-lipoprotein*
C. Alpha-lipoprotein
D. Chylomicrons
E. Pre-beta-lipoprotein
All of the following statements about LDL
are true except:
A. It delivers cholesterol to cells
B. It contains only one apoprotein*
C. It is a marker for cardiovascular disease
D. It contains apo B-100
E. 25% of its content is utilized in the liver
In electrophoresis which class of
lipoproteins will least migrate from startline:
A. Chylomicrons*
B. VLDL
C. LDL
D. IDL
E. HDL

The largest reserve of energy in body is


stored as:
A. Liver glycogen
B. Muscle glycogen
C. Triacylglycerols in adipose tissue*
D. Blood glucose
E. Muscle protein

Hormone sensitive triacylglycerol lipase is


not activated by one hormone from
following list. Point out it.
A. Insulin*
B. Glucagon
C. Epinephrine
D. Norepinephrine
E. None of above
Lesson 15 Module 1
Point out the terminal product of betaoxidation of Higher Fatty Acids (HFA) with
even number of carbon atoms:
A. Butiryl ~ SCoA
B. Malonyl ~ SCoA
C. Acetyl~SCoA*
D. Acetoacetyl ~ SCoA
E. Propionyl~SCoA

Choose the allosteric activator of acetylCoA-carboxylase (the key enzyme of HFA


synthesis):
A. Malate
B. Oxaloacetate
C. Citrate*
D. Succinate
E. Fumarate
The removal of two-carbon units from a
fatty acyl CoA involves four sequential
reactions. Which of the following reaction
sequences is correct for the pathway of oxidation:
A. Oxidation,
dehydration,
oxidation,
cleavage
B. Hydrogenation,
dehydration,
hydrogenation, cleavage
C. Dehydrogenation,
hydration,
dehydrogenation, cleavage*
D. Reduction, hydration, dehydrogenation,
cleavage
E. Reduction,
dehydration,
reduction,
cleavage
Point out the terminal product in the last
round of -oxidation of High Fatty Acids
with odd number of carbon atoms:
A. Butyryl SCoA
B. Malonyl SCoA
C. Pyruvate
D. Acetoacetyl SCoA
E. Propionyl SCoA*
The formation of the active form of a fatty
acid is endergonic process in which the ATP
energy is consumed. But there is another
necessary participant of the fatty acid
activation. Choose it:
A. Acetyl CoA
B. CoASH*
C. GTP
D. UTP
E. Succinyl CoA
Point out the cellular location of saturated
HFA synthesis:
A. Nucleus
B. Plasmolemma
C. Cytoplasm
D. Mitochondrion
E. Endoplasmic reticulum
28

Choose the products for one round of stearic


acid -oxidation:
A. 129 ATP
B. 1 Oleyl CoA, 12 ATP
C. 2 acetyl CoA, 2 FADH2, 1 ATP
D. 1 palmitoyl CoA, 1 acetyl CoA, 1
FADH2, 1 NADH*
E. 1 stearyl CoA, 1 acetyl CoA, 1 FADH2, 1
NADH
An experimental animal has been given
excessive amount of carbon labeled glucose
for a week. In what compound can the label
be found?
A. Phenylalanine
B. Methionine
C. Palmitic acid*
D. Vitamin A
E. Arachidonic acid
There is a tissue hypoxia at myocardial
ischemia in the patient. Name the process
of lipid metabolism, whose rate is reduced
in the myocardium at this state:
A. Fat lipolysis
B. Phospholipid synthesis
C. Beta-oxidation of high fatty acids*
D. Cardiolipin synthesis
E. Ketone body synthesis
Which of the following substances is
immediate precursor of acetoacetate in
pathway ketogenesis?
A. Beta-hydroxybutyrate
B. Acetoacetyl CoA
C. Beta-hydroxybutyryl CoA
D. Acetyl CoA*
E. Beta-hydroxy-beta-methylglutaryl CoA
The most important source of reducing
equivalents (NADPH) for fatty acid
synthesis in the liver is:
A. Oxidation of acetyl CoA
B. Oxidation of glucuronic acid
C. The pentose phosphate pathway*
D. Glycolysis
E. The citric acid cycle
The energy yield by stearic acid oxidation is:
A. 12
B. 38

C. 96
D. 129
E. 146*
Humans cannot achieve a NET synthesis of
glucose from C-even fatty acids due to the
inability to convert:
A. Acetyl-CoA to malonyl CoA
B. Acetyl-CoA to acetoacetate
C. Acetyl-CoA to pyruvate*
D. Oxaloacetate to pyruvate
E. Methylmalonyl-CoA to succinyl-CoA
Acetyl CoA carboxylase is key enzyme in
fatty acid synthesis. Point out the coenzyme
of this enzyme:
A. NADH
B. FADH2
C. Biotin*
D. Phosphopantetheine
E. CoASH
A microsomal enzyme system is responsible
for the formation of some unsaturated fatty
acids. Point out an enzyme of the system:
A. NADH-cytochrome b5 reductase*
B. NADH coenzyme Q reductase
C. Succinate coenzyme Q reductase
D. Cytochrome oxidase
E. Coenzyme Q-cytochrome c reductase
Which of the following statements describes
correctly ketone bodies?
A. They are accumulated in children with
fatty acid oxidation disorders
B. They are accumulated at diabetes
mellitus after insulin therapy
C. They are produced by muscle but not by
liver
D. They
include
-hydroxybutyrate,
acetoacetate and acetone*
E. They are most important nutrients for
liver
Find out the main substrates for the use by
elongase system during the formation of
stearyl CoA from palmitoyl CoA:
A. Glycerol, NADPH, palmitoyl CoA
B. Malonyl CoA, NADH, palmitoyl CoA
C. Malonyl CoA, NADPH, palmitoyl CoA*
D. Acetyl CoA, NADPH, palmitoyl CoA
E. Acetyl CoA, NADH, palmitoyl CoA
29

Point out the biological role of carnitine in


cells:
A. Antioxidant
B. Allosteric activator of enzymes
C. Transporter of fatty acid across the
mitochondrial membranes*
D. The component of respiratory chain
E. The enzyme inhibitor
Point out the substrate for acyl-CoAdehydrogenase (beta-oxidation of HFA):
A. Acetyl~SCoA
B. Enoyl~SCoA
C. Butyryl~SCoA*
D. Beta-hydroxyacyl~SCoA
E. Beta-ketoacyl~SCoA
Point out the process or reaction where
acetone is formed as end- product:
A. Beta-oxidation of HFA
B. Decarboxylation of acetoacetic acid*
C. Condensation of two acetyl-CoA
molecules
D. Synthesis of HFA
E. Decarboxylation of beta-hydroxybutyric
acid
A 1 y.o. child with symptoms of muscle
affection was admitted to the hospital.
Examination revealed carnitine deficiency in
muscles. Biochemical base of this pathology
is disturbed process of:
A. Regulation
of
Ca2+
level
in
mitochondria
B. Transporting of fatty acids to the matrix
of mitochondria*
C. Actin and myosin synthesis
D. Lactic acid utilization
E. Substrate phosphorylation
A sportsman was recommended to take a
medication that contains carnitine in order to
improve his results. What process is
activated by carnitine the most?
A. Synthesis of steroid hormones
B. Fatty acids transport to mitochondria*
C. Tissue respiration
D. Synthesis of ketone bodies
E. Synthesis of proteins

It is established, that -oxidation of high


fatty acids is carried out by multienzyme
complex in cells. Choose the enzyme that
is not the component of this complex:
A. Acyl-CoA-dehydrogenase
B. 3-Hydroxy-acyl-CoA-dehydrogenase
C. Enoyl CoA - hydratase
D. Aldolase*
E. Thiolase
It is established, that the high fatty acid
radical is lengthened in two carbon atoms
by palmitate synthetase complex action
each cycle. Point out the donor of these
two carbon atoms during the synthesis:
A. Stearyl-CoA
B. Palmityl-CoA
C. Lauryl-CoA
D. Malonyl-CoA*
E. Acetyl-CoA
Point out the ketone body that is not utilized
in human organism:
A. Acetoacetate
B. Beta-hydroxybutyrate
C. Acetone*
D. All the substances proposed
E. None of the substances proposed
Which of the following steps is involved in
the formation of glucose from lipolysis
product?
A. 2 glycerols from lipolysis are taken up
by liver cells and dimerized to fructose
B. Glycerol from lipolysis is converted to
triacylglycerols
C. 2 glycerols from lipolysis are
phosphorylated, converted in a few steps to
fructose-1,6-bisphosphate, and eventually
converted to glucose*
D. Fatty acids from lipolysis are oxidized,
producing
FADH2
and
stimulating
gluconeogenesis
E. Fatty acids from lipolysis are converted
to glucose
In uncontrolled diabetes mellitus,
acetoacetic acid and beta-hydroxybutyric
acid are produced in:
A. Pancreas
B. Liver*
C. Small intestine
30

D. Kidneys
E. Brain
What compound production and utilization
become more significant during starvation?
A. Triacylglycerols
B. Ketone bodies*
C. Glycogen
D. Fatty acids
E. Uric acid
Which of the following organs (tissues)
cannot use ketone bodies:
A. Brain
B. Liver*
C. Kidney
D. Myocardium
E. Skeletal muscle
Lesson 16
Module 1
Point out the drug used for the decrease of
cholesterol level in the blood of patients allosteric inhibitor for key enzyme of
cholesterol synthesis:
A. Aspirin
B. Lovastatin*
C. Barbiturate
D. Indomethacin
E. Antimicin A
Point out the atherogenic lipoproteins:
A. Nascent chylomicrones
B. Fatty acid-albumins complex
C. Low density lipoproteins*
D. High density lipoproteins
E. Remnant chylomicrones
The conversion of cholesterol catalyzed by
7-hydroxylase is inhibited by:
A. Cortisol
B. Bile acids*
C. Testosterone
D. Calcitriol
E. Glycine and taurine
All hereinafter stated compounds are
products of cholesterol catabolic conversion
except one. Choose it:
A. Glucuronic acid*
B. Cholic acid
C. Chenodeoxycholic acid
D. Glucocorticoids

E. Mineralocorticoids
Find out the coenzyme used in some
reactions of cholesterol synthesis:
A. FAD
B. NADH
C. Pyridoxal phosphate
D. NADPH*
E. Biotin
Which statement is not associated with
situation of fat deposition?
A. Insulin secretion is increased
B. Blood free fatty acid levels are elevated
C. Hormone sensitive triacylglycerol lipase
activity is increased*
D. Blood VLDL and chilomicrone levels
are elevated
E. Lipoprotein lipase activity is increased
Which of the following statements explains
correctly metabolic alterations that are
specific for persons disposed to obesity
beside people having standart weight?
A. Coupling of respiration with oxidative
phosphorylation is much more
B. There is any genetic defect in leptin*
C. Rate of Krebs cycle reactions is higher
D. Calorie intake is much less
E. Rate of fatty acid -oxidation is much
less
The patient with diabetes mellitus has been
delivered in hospital in the state of
unconsciousness. Arterial pressure is low.
The patient has acidosis. Point substances,
which accumulation in the blood results in
these manifestations:
A. Ketone bodies*
B. Monosaccharides
C. Cholesterol esters
D. High fatty acids
E. Amines
All of the following statements regarding
hypercholesterolemia
(type
IIa
hyperlipidemia) are correct except:
A. There is an increased risk of coronary
artery disease
B. The blood serum cholesterol levels are
increased

31

C. The blood serum triacylglycerol levels


are elevated*
D. The blood serum low density lipoprotein
(LDL) levels are high
E. It is due to a deficiency of LDL receptors
The liver is major site of cholesterol
biosynthesis, although other tissues are also
active in this regard.
The terminal products of cholesterol
catabolism in the liver play the important
role during lipids intake in human organism
Point out these products:
A. Catecholamines
B. Bile acids*
C. Corticosteroids
D. Acetyl - SCoA
E. Cholanic acid
The synthesis of 1,25-dihydroxycholecalciferol
take place:
A. In the skin under action of ultraviolet
light from 7-dehydrocholesterol
B. In the intestine from cholecalciferol
C. In the liver from cholecalciferol
D. In the kidney from 25hydroxycholecalciferol*
E. It is not synthesized in mammals
Find out pathological state associated with
hypercholesterolemia in patient:
A. Hypothyroidism
B. Non-insulin dependent diabetes mellitus
C. Cholestasis
D. Nephritic syndrome
E. All the positions are right*
The inhibition of one liver enzyme in
cholesterol synthesis was after surplus
intake of fat meat. Name this enzyme:
A. Beta-hydroxy-beta-methyl-glutaryl-CoAreductase*
B. Mevalonate kinase
C. Acetyl - CoA- acetyl transferase
D. Isopentenyl pyrophosphate isomerase
E. Squalene oxygenase
The enzyme system for cholesterol
biosynthesis is located in the:
A. Golgi apparatus
B. Cytosol and EPR (smooth part)*
C. Cytosol and mitochodrial membrane

D. Mitochodrial matrix
E. Plasma membrane
Choose the value of Cholesterol (LDL) +
Cholesterol (VLDL)/ Cholesterol(HDL) ratio
which is peculiar to healthy people:
A. 0.5-1.0
B. More then 3.5
C. Less then 3.5*
D. 4.0-6.0
E. 7.0-8.0
Choose the key metabolite that combines
carbohydrates catabolic pathways with
anabolic processes in lipids metabolism:
A. Serine
B. Alanine
C. Acetyl~SCoA*
D. Mevalonate
E. Pyruvate
What plasma lipoproteins, whose levels rise,
are inversely correlated with
atherosclerosis? :
A. Nascent chylomicrones
B. Fatty acid-albumins complex
C. High density lipoproteins*
D. Low density lipoproteins
E. Remnant chylomicrones
Cholesterol is the precursor for synthesis of
all pointed below hormones except:
A. Cortisol
B. Aldosterone
C. Progesterone
D. Testosterone
E. Insulin*
Find out the vitamin derivative that is
synthesized from cholesterol in humans:
A. Progesterone
B. 1,25-dihydroxycholecalciferol*
C. Estradiol
D. Cholesterol ester
E. Testosterone
The main chemical compounds
cholesterol synthesis are:
A. Acetyl CoA, NADPH, ATP*
B. Malonyl CoA, NADPH, ATP
C. Acetyl CoA, GTP, pantothenate
D. Acyl CoA, carnitine, ATP

for

32

E. Glycerol, phosphate, choline


Kidney Examination of cell culture got from
a patient with lysosomal pathology revealed
accumulation of great quantity of lipids in
the lysosomes. What of the following
diseases is this disturbance typical for?
A. Gout
B. Phenylketonuria
C. Wilson disease
D. Tay-Sachs disease*
E. Galactosemia
Point out the hormone that decreases the rate
of lipolysis in adipose tissue:
A. Epinephrine
B. ACTH
C. Somatotropin
D. Insulin*
E. Glucagon
Choose the factor that can cause the 7dehydrocholesterol conversion into vitamin
D3 (in skin, only):
A. The accumulation of cholesterol
B.Cytochrome P450
C. Hydroxylase action
D. UV-light*
E. Low levels of oxygen in the blood
Choose the incorrect statement related to
biological role of cholesterol:
A. It is a structural component of membrane
B. It is an energy source for cells*
C. It is precursor for bile salts
D. It is precursor for glucocorticoids
E. It is precursor for sex hormones
The analysis of the blood of the patient
has revealed the considerable growth of
VLDL and autoimmune LDL levels.
Specify the pathology that can be in the
patient:
A. Acromegaly
B. Phenylketonuria
C. Diabetes insipidus
D. Atherosclerosis*
E. Viral hepatitis
In Niemann-Pick`s disease the following
substances accumulate in CNS in excess
levels:

A.
B.
C.
D.
E.

Triacylglycerols
Lysophosphingosides
Phosphoinositides
Sphingomyelins*
Cholesterol esters

One of the following is scavenger of


cholesterol from tissue and prevents
atherosclerosis:
A. VLDL
B. LDL
C. Chylomicrons
D. HDL*
E. Carnitine
Gaucher`s disease is due to the accumulation
of:
A. Glucose
B. Glucocerebroside*
C. Galactose
D. Sphingomyelin
E. Triacylglycerol
Liebermann-Burchardt reaction is used for
the identification of :
A. Galactose
B. Glucose
C. Cholesterol*
D. Prostaglandins
E. Urea
The best marker for dislipidemia is:
A. LDL/HDL*
B. Serum cholesterol
C. Cholesterol/TG
D. Blood cholesterol
E. Cholesterol/HDL
Which of the following is found in
conjugation with bile acids:
A. Cholic acid
B. Pregnenolone
C. Glycine*
D. Cholyl acetyl-CoA
E. Hydrochloric acid
The rate limiting enzyme in bile acid
synthesis is:
A. HMG-Co-A reductase
B. Decarboxylase
C. HMG-CoA synthetase
D. 7-Alpha hydroxylase*
E. 7-Beta hydroxylase

33

A 12-year old patient was found to have


blood serum cholesterol at the rate of 25
mmol/l. The boy has a history of hereditary
familial hypercholesterolemia, which is
caused by the impaired synthesis of the
following protein receptors to:
Low density lipoproteins*
High density lipoproteins
Intermediate density lipoproteins
Very low density lipoproteins
Chylomicrons
A 67-year old male patient consumes eggs,
pork fat, butter, milk and fatty meal. Blood
test results: cholesterol 12,3 mmol/l, total
lipids 8,2 g/l, increased low-density
lipoprotein fraction (LDL). What type of
hyperlipoproteinemia is observed in the
patient?
A.
B.
C.
D.
E.

Cholesterol, hyperlipoproteinemia
Hyperlipoproteinemia type IIa*
Hyperlipoproteinemia type I
Hyperlipoproteinemia type IIb
Hyperlipoproteinemia type IV

One of the factors that cause obesity is the


inhibition of fatty acids oxidation due to:
A. Choline deficiency
B. Impaired phospholipids synthesis
C. Low level of carnitine*
D. Lack of carbohydrates in the diet
E. Excessive consumption of fatty
foods
A patient underwent a course of treatment
for atherosclerosis. Laboratory tests revealed
an increase in the anti-atherogenic
lipoprotein fraction in the blood plasma. The
treatment efficacy is confirmed by the
increase in:
A. HDL *
B. LDL
C. VLDL
D. Chylomicrons
E. IDL
Lesson17 Module 1
Glutamate decarboxylation results in the
formation of inhibitory transmitter in CNS.
Name it:
A. Glutathione

B.
C.
D.
E.

Gamma amino butyric acid*


Serotonin
Histamine
Asparagine

Which of the following statements related to


biological role of transamination is wrong?
A. Redistribution of amino groups
B. Production of non-essential amino acids
C. Formation of biogenic amines*
D. Divergence the excess amino acids
towards energy generation
E. Collection finally nitrogen in glutamate
for subsequent deamination and urea
synthesis
Point out the liver enzyme, which takes part
in second step of transdeamination of amino
acid:
A. Alpha-ketoglutarate dehydrogenase
B. Glutamate dehydrogenase*
C. Glutamate decarboxylase
D. L-Alanine oxidase
E. Tryptophan hydroxylase
Direct deamination is possible for two
amino acids due to dehydratases (because of
those amino acids side chain hydroxyl group
is a good leaving group). Point out these
amino acids:
A. Glutamate, aspartate
B. Serine, threonine*
C. Phenylalanine, tyrosine
D. Cysteine, methionine
E. Arginine, histidine
Human Glutamate dehydrogenase is
predominantly located in the:
A. Cytosol
B. Inner mitochondrial membrane
C. Outer mitochondrial membrane
D. Mitochondrial matrix*
E. Endoplasmic reticulum
Choose the enzyme, whose genetic defect
results in the GABA (Gamma-Amino
Butyric Acid) levels decrease in the brain:
A. Tryptophan decarboxylase
B. Phenylalanine hydroxylase
C. Histidine decarboxylase
D. Alanine hydroxylase
E. Glutamate decarboxylase*
34

E. Glutamate dehydrogenase*
Which of the following coenzymes serves as
a non-protein part of amino acid
decarboxylase?
A. Pyridoxal phosphate*
B. Coenzyme A
C. Biotin
D. NADH
E. NADPH
The carbon skeletons of amino acids finally
have one or more of the following fates
except:
A. Oxidation via Krebs cycle to produce
energy
B. Synthesis of glucose
C. Synthesis of urea*
D. Formation of fatty acid and ketone
bodies
E. Synthesis of non-essential amino acids
The coenzyme that participates
transamination reaction is:
A. Coenzyme A
B. Coenzyme Q
C. Pyridoxal phosphate*
D. Thiamine pyrophosphate
E. NADPH

in

Norepinephrine is produced (in mammals)


from:
A. Tryptophan
B. Tyrosine*
C. Pyruvate
D. Arginine
E. Tryptamine
The entry point into the citric acid cycle for
isoleucine, valine, and the product of oddchain fatty acids beta-oxidation is:
A. Pyruvate
B. Oxaloacetate
C. Citrate
D. Succinyl CoA*
E. Fumarate
The most important enzyme involved in
oxidative deamination is:
A. L-amino acid oxidase
B. D-amino acid oxidase
C. Amino acid dehydrase
D. Histidase

Most amino acids undergo transamination to


concentrate finally nitrogen in two amino
acids, only, for subsequent urea formation.
Choose these amino acids:
A. Lysine, proline
B. Glutamate, aspartate*
C. Alanine, phenylalanine
D. Serine, tyrosine
E. Arginine, histidine
Glutamate dehydrogenase is controlled by
allosteric regulation. Point out the inhibitors
of this enzyme:
A. ATP, GTP*
B. ADP, GDP
C. AMP, ADP
D. NAD+, NADP+
E. NADH, NADPH
Which of the following descriptions related
to serotonin pathway isnt correct?
A. Serotonin synthesis lowering in the brain
causes depression
B. The largest amount of serotonin is
synthesized in the intestinal cells*
C. The first enzyme of the pathway is
tetrahydrobiopterin-dependent hydroxylase
D. The second enzyme of the pathway is
biotin-dependent aromatic amino acid
decarboxylase
E. Serotonin is synthesized from
thyptophan
Name the ketogenic amino acids:
A. Leucine, lysine*
B. Alanine, arginine
C. Glutamate, aspartate
D. Cysteine, glycine
E. Arginine, asparagine
D-amino acid oxidase is important for the
conversion of unnatural (for human body but
regularly taken in the diet from plants) D
amino acids to L-amino acids. What
prosthetic group is non-protein part of the
enzyme?
A. NADH
B. NADPH
C. TPP
D. FAD*
35

E. Pyridoxal phosphate
Point out the enzyme, whose activity is
determined in the blood plasma during the
unicteric period of viral hepatitis:
A. Phenylalanine hydroxylase
B. Creatine phosphokinase
C. Glutamate dehydrogenase
D. Alanine transaminase*
E. Ornithine carbomoylphoshate transferase
Choose the enzyme of the blood plasma
whose activity increases in ten or more
times for 3-4 hours after myocardium
infarction:
A. Alanine transaminase
B. Aspartate transaminase*
C. Alkaline phoshatase
D. Arginase
E. Leucine aminopeptidase
Point out the vitamin, whose deficiency
causes the violations in the transamination
and decarboxylation of amino acids:
A. Vitamin C
B. Vitamin B1
C. Vitamin B2
D. Vitamin B9
E. Vitamin B6*
Utilization of amino acids from the body
pool is possible in all following ways except
one. Choose it:
A. Porphyrins production
B. Purines production
C. Biogenic amine formation
D. Glucose and ketone bodies synthesis
E. Pyridoxal phosphate synthesis*
Point out the glucogenic amino acids:
A. Glutamate
B. Alanine
C. Serine
D. Aspartate
E. All the positions are right*
Point out the liver enzyme catalyzing the
reversible oxidative deamination:
A. Glutamate dehydrogenase*
B. Alanine transaminase
C. Monoamino oxidase

D. Aspartate transaminase
E. Arginase
In course of histidine catabolism a biogenic
amine is formed that has powerful
vasodilatation effect. Name it:
A. Noradrenalin
B. Dioxyphenylalanine
C. Serotonin
D. Dopamine
E. Histamine*
According to clinical indications a patient
was administered pyridoxal phosphate. What
process is this medication intended to
correct?
A. Deamination of purine nucleotide
B. Synthesis of purine and pyrimidine
bases.
C. Transamination and decarboxylation of
amino acids*
D. Protein synthesis
E. Oxidative decarboxylation of ketoacids
A patient diagnosed with carcinoma of
bowels was admitted to the hospital.
Analysis revealed high production of
serotonin. It is known that this substance is
formed from 5-hydroxy tryptophan amino
acid.
What
biochemical
mechanism
underlies this process?
A. Formation of paired compounds
B. Decarboxylation*
C. Transamination
D. Microsomal oxidation
E. Desamination
The tested enzyme is related to the class
of oxidoreductases and contains the
coenzyme NADPH. It takes part in the
oxidative deamination of one of the amino
acids and in the reductive amination of
alphaketoglutarate. Point out it:
A. Glutamate dehydrogenase*
B. Alanine transaminase
C. D-Alanine oxidase
D. Aspartate transaminase
E. Arginase
Immediate products of pyruvate metabolism
(using one reaction only) are all except:
A. Acetyl-CoA
B. Alanine

36

C. Lactate
D. Oxaloacetate
E. 2-Phosphoglycerate*

An inhibitory mediator is formed by the


decarboxylation of glutamate in the CNS.
Name it:
A. Asparagine
B. Serotonine
C. Histamine
D. GABA *
E. Glutathione
It is known that the monoamine oxidase
(MAO) enzyme plays an important part in
the metabolism of catecholamine
neurotransmitters. In what way this enzyme
inactivates these neurotransmitters
(norepinephrine, epinephrine, dopamine)?
A.
B.
C.
D.
E.

Oxidative deamination *
Carboxylation
Addition of an amino group
Removal of a methyl group
Hydrolysis

Lesson 18
Module 1
Choose the type of infringement in
pathology named alkaptonuria:
A. Genetic deficiency of enzyme*
B. The inhibition of amino acid formation
C. The renal insufficiency
D. The damage of receptor synthesis
E. The viral damage of hepatocyte

Point out the cofactor required for the


tyrosine formation from phenylalanine:
A. Coenzyme A
B. Tetrahydrobiopterin*
C. Coenzyme Q
D. Tetrahydrofolate
E. Lipoic acid
Which of the following compounds is not a
catabolic product of tryptophan?
A. Kynurenine
B. Serotonin
C. Melanin *
D. Melatonin
E. Tryptamine
The child has the genetic defect of one
enzyme. It was proved by the appearance
of phenylpyruvate in the urine. Point out
this enzyme:
A. Tyrosine hydroxylase
B. Phenylalanine 4-monooxygenase*
C. 5-tryptophan hydroxylase
D. Glycine amidase
E. Proline hydroxylase
Name an amino acid which is precursor in
the synthesis of some hormones in human
body:
A. Lysine
B. Alanine
C. Arginine
D. Serine
E. Tyrosine*

One of hereinafter stated compounds forms


a connecting link between urea cycle and
Krebs cycle via oxaloacetate. Point out it:
A. Alanine
B. Phenylalanine
C. Tryptophan
D. Aspartate*
E. Glutamate

The major source of ammonia in the kidneys


is:
A. Glutamate
B. Aspartate
C. Glutamine*
D. Histidine
E. Urea

Point out the pathological state that may be


estimated in patients with genetic defect of
Phenylalanine -4-hydroxylase:
A. Glucosemia
B. Hyperuricemia
C. Aminoaciduria*
D. Hypercholesterinemia
E. Dislipoproteinemia

Try to choose the index of blood plasma


used for estimation of liver function in
utilization of ammonia:
A. Pyruvic acid content
B. Urea content*
C. Uric acid concentration
D. Glucose concentration
E. Ketone bodies content

37

Which of the following statements is


incorrect
concerning
synthesis
of
carbamoyl phosphate?
A. Mitochondrial carbamoyl phosphate
synthase requires aspartate for its activity*
B. Two ATP may be consumed for its
synthesis
C. This synthesis is catalyzed by carbamoyl
phosphate synthase
D. It is the key reaction for urea formation
E. The nitrogen source can be either
ammonia or glutamine
Cerebral trauma caused the increase of
ammonia formation. What amino acid takes
part in removal of ammonia from cerebral
tissue?
A. Tryptophan
B. Lysine
C. Glutamic acid*
D. Valine
E. rosine
The amino acids required for the formation
of glutathione are:
A. Glycine, cysteine, glutamate*
B. Serine, Tyrosine, tryptophan
C. Valine, leucine, isoleicine
D. Glutamate, aspartate, arginine
E. Cysteine, methionine, proline
Albinism is due to the lack of special
enzyme in melanocytes. Name it:
A. Tyrosine decarboxylase
B. Tyrosinase*
C. Homogentisate oxydase
D. Kynureninase
E. Phenylalanine hydroxylase
Which one of the following amino acids
may be converted to an intermediate
metabolite of either the Krebs cycle or Urea
cycle?
A. Tyrosine
B. Lysine
C. Leucine
D. Tryptophan
E. Aspartate*
Alkaptonuria is due to the deficiency of one
enzyme in human tissues. Choose it
A. Tyrosinase

B. Phenylalanine hydroxylase
C. Dihydroxyphenylalanine
decarboxylase
D. Homogentisate oxidase*
E. DOPA oxidase

(DOPA)

Citrullinemia accompanied with vomiting,


mental retardation, convulsions. Which urea
cycle enzyme hereditary defect is a cause of
mentioned disturbances?
A. Arginase
B. Arginosuccinase
C. Arginosuccinate synthase*
D. Carbamoyl phosphate synthase I
E. Ornithine transcarbamoylase
Point out the index of the blood plasma that
may be used for the estimation of liver
parenchyma damage:
A. Glucose
B. Urea
C. Free amino acids
D. Cholesterol ester
E. Ca2+
Point out the coenzyme used for
hydroxylases structure in phenylalanine and
tyrosine conversions:
A. Tetrahydrobiopterin*
B. Dihydrobiopterin
C. NADPH
D. FAD
E. Biotin
Which of the following functions is not
specific for glutathione:
A. It serves as coenzyme for certain
enzymes e.g. prostaglandine PGE2 synthase
B. It prevents the oxidation of sulfhydryl
groups of several proteins to disulfide
groups
C. It serves as a storage and transport form
of NH3*
D. It protects a cell against cytotoxic H2O2
being coenzyme of glutathione peroxidase
E. It is involved in the transport of amino
acids across cellular membrane in the
intestine and kidney
Point out the pathology that is estimated in
patients with genetic deficiency of
dihydrobiopterin reductase:
38

A. Classic phenylketonuria
B. Gout
C. Phenylketonuria type II*
D. Diabetes mellitus
E. Ishemic heart disease
Find out the main important pathway for
ammonia utilization in the brain:
A. Conversion into glucose
B. Urea formation
C. Amino acid decarboxylation
D. Synthesis of glutamine from alphaketoglutarate*
E. Ammonia salts formation
Choose the hormone that is formed from
tryptophan:
A. Epinephrine
B. Thyroxin
C. Nor-epinephrine
D. Melatonin*
E. Histamine
Point out the enzyme whose deficiency
causes the hyperammonemia state in patient:
A. Ornithine transcarbamoylase*
B. Arginine transaminase
C. Pyruvate dehydrogenase
D. Aspartate transaminase
E. L-Alanine oxidase
Which of the following statements is
incorrect
concerning
synthesis
of
carbamoyl phosphate?
A It is mediated by different enzymes in
the cytosol and mitochondria of the liver
B The nitrogen source can be either
ammonia or glutamine
C It is the key reaction for urea formation
D This synthesis is catalyzed by ornithine
transcarbamoylase*
E Two ATP may be consumed for its
synthesis
N-acetylglutamate is activator for one
enzyme involved in urea formation. Choose
it:
A. Carbamoyl phosphate synthase I*
B. Ornithine transcarbamoylase
C. Argino-succinate synthase
D. Argino-succinase
E. Arginase

Nappies of a newborn have dark spots that


witness the presence of homogentisic acid
oxidation product. Choose the substance
whose metabolic disorder is associated with
accumulation of homogentisic acid in the
organism:
A. Cholesterol
B. Galactose
C. Tyrosine*
D. Tryptophan
E. Methionine
A 4 y.o. boy has had recently serious viral
hepatitis. Now there are such clinical
symptoms as vomiting, unconsciousness,
fits. There is hyperammoniemia in patient,
too. Disturbance of which biochemical
process caused such pathological condition
of the patient?
A. Increased putrefaction of proteins in
bowels
B. Inhibition of transamination enzymes
C. Disturbed neutralization of ammonia in
the liver*
D. Activation of amino acid decaboxylation
E. Disturbed neutralization of biogenic
amines
A 4 y.o. child with signs of durative
proteinic starvation was admitted to the
hospital. The signs were as follows: Growth
inhibition,
anemia,
oedema,
mental
deficiency. Choose the cause of oedema
development:
A. Reduced synthesis of lipoproteins
B. Reduced synthesis of glycoproteins
C. Reduced synthesis of hemoglobin
D. Reduced synthesis of globulins
E. Reduced synthesis of albumins*
Albinos can't stand sun impact they don't
require sun-tan but get sunburns. Disturbed
metabolism of what amino acid underlies
this phenomenon?
A. Histidine
B. Phenylalanine*
C. Tryptophan
D. Glutamic acid
E. Methionine

39

Maple syrup urine disease is due to a defect


in the enzyme complex which catalyzes
oxidative decarboxylation reaction and is
similar to any other -keto acid
dehydrogenase complexes in composition.
Some patients who exhibit the symptoms of
this disease respond to therapeutic doses of
hereinafter stated vitamins. Point out it:
A. Thiamine*
B. Pyridoxine
C. Biotin
D. Cobalamine
E. Ascorbic acid
After a serious viral infection a 3-year-old
child has repeated vomiting, loss of
consciousness, convulsions. Examination
revealed hyperammoniemia. What may have
caused changes of biochemical blood
indexes of this child?
A. Activated processes of amino acids
decarboxylation
B. The increased putrefaction of proteins in
intestines
C. The inhibited activity of enzymes for
transamination
D. Disorder of ammonia neutralization in
ornithine cycle*
E. Disorder
of
biogenic
amines
neutralization
The brain ammonia isnt neutralized
through the formation of urea. Name the
terminal
product
of
ammonia
neutralization in the brain:
A. Urea
B. Glutamine*
C. Carbamoyl phosphate
D. Aspartic acid
E. Alanine
Phenylketonuria is due to the absence of
certain enzyme. Choose the reaction
catalyzed by this enzyme:
A. Homogentisic acid oxidation
B. Phenylalanine hydroxylation*
C. Tyrosine deamination
D. Tyrosine iodination
E. Conversion
of
tyrosine
to
dihydroxyphenylalanine (DOPA)

Choose the blood plasma index that is


used in screening of newborn for
phenylketonuria estimation:
A. Phenylalanine*
B. Acetoacetate
C. Uric acid
D. Dihydroxyphenylalanine
E. Pyruvate
Point out the vitamin derivative that may
be synthesized from tryptophan:
A. FAD
B. CoASH
C. FMN
D. NADH*
E. Carboxybiotin
A boy (of 10 years) complains of general
weakness, dizziness, and tiredness. A
mental retardation is observed. A
concentration of valine, leucine, isoleucine
is high in blood and urine. Urine has a
specific odour. Name the probable
diagnosis:
A. Basedows disease
B. Histidinemia
C. Maple syrup urine disease*
D. Addisons disease
E. Tyrosinemia
Three biogenic amines are formed during the
metabolism of tyrosine. Point out one of
them:
A. Thiamine
B. Serotonin
C. Histamine
D. Thyroxin
E. Adrenalin*
A newborn child was found to have reduced
intensity of sucking, frequent vomiting,
hypotonia. Urine and blood exhibit
increased concentration of citrulline. What
metabolic process is disturbed?
A. Ornithinic (Urea) cycle*
B. Cori cycle
C. Glycolysis
D. Tricarboxylic acid cycle
E. Glyconeogenesis
Albinism is due to deficiency of the
following enzyme:
40

A.
B.
C.
D.
E.

Phenylalanine hydroxylase
Tyrosinase*
Homohentisic acid oxidase
Tyrosine decarboxylase
Tyrosine transaminase

Maple syrup urine disease is due to


deficiency of:
A. Decarboxylase
B. Dehydroxylase
C. Transaminase
D. Deaminase
E. Alpha-keto
acid
dehydrogenase
complex*

Urea is produced by single enzyme. Point


out it:
A. Urease
B. Uricase
C. Arginase *
D. Glutaminase
E. Carbamoyl phosphate synthetase
Glycine is used for synthesis of:
A. Hemoglobin
B. Glutathione
C. Purine
D. Creatine
E. All the proposed options are correct*

Xanthurenic acid is a product of metabolism


of:
A. Tryptophan*
B. Glucuronic asid
C. Xanthine
D. Uric acid
E. Urea
If urine sample darkens on air standing,
most likely diagnosis for patient is:
A. Phenylketonuria
B. Alkaptonuria*
C. Maple syrup urine desease
D. Diabetes mellitus
E. Albinism
Melanin may be synthesized from:
A. Tryptophan
B. Phenylalanine*
C. Cysteine
D. Methionine
E. Serine
The urine of patient with alkaptonuria
contains:
A. Homogentisic acid*
B. Phenylalanine
C. Ketones
D. Acetates
E. None of the above
The most direct precursor of taurine is:
A. Glycine
B. Cysteine*
C. Methionine
D. Glutathione
E. Tyrosine
41

Vous aimerez peut-être aussi